🔙 Quay lại trang tải sách pdf ebook Tạp Chí Pi Tập 1 số 3
Ebooks
Nhóm Zalo
Tập 1 Số 3 Tháng 3-2017 Tạp chí Pi
[toankho.com]: Sharing nice and hard problems. Get more books at http://book.toankho.com
Tập 1 Số 3 Tháng 3-2017 Tạp chí Pi
TRONG SỐ NÀY
Đồng dư
Ngô Bảo Châu
Tiền mật mã
Phạm Huy Điển
Bài toán Brachistochrone: từ lời thách đố đến điều khiển tối ưu Nguyễn Bảo Huy
Một số bài toán Số học - Tổ hợp
Hà Huy Khoái
Thách thức Toán học
Số Pi – cuộc chơi thú vị của người yêu toán học
Nguyễn Thanh Giang
Lê Văn Thiêm, con người và sự nghiệp
Hà Huy Khoái
Giới thiệu giải thưởng Lê Văn Thiêm
Hà Huy Khoái
Giới thiệu Trại hè Toán và Khoa học MaSSP
Lê Hồng Nhung và Đỗ Thị Thu Thảo
Nói chuyện với GS Đàm Thanh Sơn
Ngô Bảo Châu (thực hiện)
APMO dành cho học sinh tiểu học và THCS (APMOPS) Trần Nam Dũng
Thế nào là một năm?
Phạm Vũ Lộc
Quán Toán
Toán của Bi
Thể lệ gửi bài và đặt báo
Tập 1 Số 3 Tháng 3-2017 Tạp chí Pi
ĐỒNG DƯ
Ngô Bảo Châu
Đồng dư là một khái niệm Toán học cơ bản, đơn giản và sơ cấp. Nó thường được giảng dạy ngay từ cấp THCS. Đó cũng là khuôn khổ để phát biểu và chứng minh một trong những định lý Toán học thực sự đầu tiên mà học sinh được học, đó là định lý Fermat nhỏ. Lần đầu tiên tôi được nghe giảng về định lý Fermat nhỏ có lẽ là vào thời gian học lớp 6, đó cũng là lần đầu tiên tôi nhận ra được cái mà người ta gọi là vẻ đẹp Toán học, một thứ rất khó định nghĩa, nhưng cũng là cái mà bao nhiêu người vẫn miệt mài, thầm lặng theo đuổi.
Mục đích của bài viết nhỏ này là thuật lại sự xuất hiện của khái niệm đồng dư từ nền văn minh cổ đại, và rồi nhấn mạnh một khía cạnh mà có lẽ ít người để ý đến, đó là tính xúc tác của nó trong sự hình thành các cấu trúc đại số trong Toán học hiện đại.
1. Euclid và Sunzi
Chương VI và IX của quyển sách "Cơ sở" của Eudlid có nhắc đến tính chất chia hết, khái niệm số chẵn và số lẻ, nhưng chưa phát biểu tường minh khái niệm đồng dư.
Hai số nguyên a1, a2 được coi là đồng dư theo modulo b nếu a1 − a2 chia hết cho b, hay nói cách khác a1 và a2 có cùng một phần dư Euclid trong phép chia cho b. Ta ký hiệu
a1 ≡ a2 (mod b).
Khái niệm đồng dư xuất hiện khá rõ nét trong định lý phần dư Trung Hoa. Người Trung Hoa gọi nó là bài toán Hàn Tín điểm binh: Một nhóm không quá một trăm binh lính xếp hàng bảy thì dư ra một người, xếp hàng năm thì dư ra ba người, xếp hàng ba thì không dư ra ai.
Tướng quân giỏi nhẩm sẽ tính ra rằng số binh bằng đúng bảy mươi tám.
Dựa theo tra cứu thì thấy định lý này được phát biểu lần đầu trong sách Toán pháp Tôn Tử (thế kỷ thứ III − V sau Công nguyên), không liên quan gì đến Binh pháp Tôn Tử (thế kỷ thứ V trước Công nguyên). Vì vậy có thể
đặt câu hỏi gán ghép bài toán đồng dư với tên của Hàn Tín có phải là một nhầm lẫn lịch sử hay không?
Định lý phần dư Trung Hoa được phát biểu với ký hiệu hiện đại như sau:
Cho m là số nguyên dương. Nếu n1, n2, ..., nm là các số nguyên đôi một nguyên tố cùng nhau và r1, r2, ..., rm là các số nguyên thoả mãn 0 ≤ ri < ni với i ∈ {1, 2, ..., m} thì tồn tại duy nhất một số nguyên r thoả mãn và r ≡ ri (mod ni),với mọi i ∈ {1,2,...,m}.
Giả thiết quan trọng trong định lý phần dư Trung Hoa là giả thiết nguyên tố cùng nhau. Hai số nguyên a và b được coi là nguyên tố cùng nhau nếu như ước số chung lớn nhất của
Tập 1 Số 3 Tháng 3-2017 Tạp chí Pi 1
chúng bằng 1. Nếu a và b là hai số nguyên dương, ta ký hiệu gcd(a,b) là ước số chung lớn nhất của chúng. Như vậy, a và b là nguyên tố cùng nhau khi và chỉ khi gcd(a,b) = 1.
Sách Cơ sở của Euclid, chương VI, có khảo sát khá kỹ khái niệm nguyên tố cùng nhau.
Eculid
Mệnh đề số 2 trong chương VI đưa ra một thuật toán hiệu quả cho việc tìm ước số chung lớn nhất của hai số nguyên dương và đồng thời xác định xem hai số nguyên cho trước có nguyên tố cùng nhau hay không.
Cho a và b là hai số nguyên dương. Xét (x, y) là một cặp biến nguyên với giá trị ban đầu là (x, y) = (a, b). Ta thực hiện phép chia Euclid x = yq+r, với q,r ∈ ℤ, 0 ≤ r < y. Nếu r = 0, ước số chung lớn nhất của x và y bằng y. Nếu r > 0, ta thực hiện vòng lặp với giá trị mới (x, y) = (b, r)
thay cho cặp số (x, y) = (a, b) ban đầu. Thuật toán dừng khi mà phép chia Euclid không còn phần dư. Thuật toán hiển nhiên phải dừng sau một số hữu hạn bước vì b > r và biến y không thể giảm mãi trong tập các số nguyên dương.
Để giải thích cho thuật toán Euclid, ta cần nhận xét rằng gcd(a, b) = gcd(b, r), tức là ước số chung lớn nhất của cặp số (a, b) đúng bằng ước số chung lớn nhất của cặp số (b, r). Thật vậy, từ quan hệ a = bq+r, ta rút ra rằng một số nguyên d là ước của cả a và b khi và chỉ khi nó là ước của cả b và r. Ở bước cuối cùng, khi mà phần dư bằng 0, hiển nhiên ước số chung lớn nhất của x và y đúng bằng y.
Một nhận xét rất quan trọng là trong tiến trình của thuật toán Euclid với cặp số biến thiên (x, y) = (a, b) ở bước thứ nhất, rồi (x, y) = (b, r) ở bước thứ hai, các giá trị nhận được của x và y luôn là các tổ hợp tuyến tính nguyên của a và b, hay nói cách khác, ở mọi thời điểm, giá trị của x và **y luôn có dạng ma+bn, với m, n là những số nguyên nào đó. Ta có thể rút ra kết luận rằng ước số chung lớn nhất gcd(a, b) bắt buộc phải có dạng này, tức là tồn tại những số nguyên m, n sao cho
gcd(a,b) = ma + mb (1)
Đây cũng chính là tổ hợp tuyến tính có giá trị tuyệt đối dương nhỏ nhất của hai số nguyên dương a**, b cho trước. Trong ngôn ngữ Đại số hiện đại, ta sẽ nói rằng tập hợp các số ở
dạng ma + nb, với m,n ∈ ℤ, là ideal sinh bởi a và b. Ideal này là ideal chính, nó có phần tử sinh là ước số chung lớn nhất gcd(a, b). Tất nhiên, ngày xưa, Euclid không có khái niệm ideal chính trong hành trang lý thuyết của mình, nhưng khái niệm này đã tiềm ẩn trong thuật toán mà ông nghĩ ra.
Cũng cần bổ sung thêm rằng chứng minh định lý phần dư Trung Hoa cho hai số nguyên n1,n2 nguyên tố cùng nhau, có thể qui về việc chỉ ra hai số nguyên m1,m2 thoả mãn m1n1 + m2n2 = 1.
2. Fermat và Euler
Nếu p là một số nguyên tố và a là một số nguyên dương không chia hết cho p thì ta có đồng dư thức
ap−1 ≡ 1 (mod p).
Đây là phát biểu của định lý Fermat nhỏ, một trong những định lý quan trọng nhất trong Số học sơ cấp.
Euler mở rộng định lý của Fermat cho trường hợp đồng dư modulo một số nguyên không nhất thiết nguyên tố. Nếu n là một số nguyên dương, ta ký hiệu φ(n) là số các số nguyên a với 0 < a ≤ n sao cho (a, n) = 1. Với một số
nguyên tố, ta có φ(p) = p−1. Số φ(n) được gọi là chỉ số Euler của n. Định lý Fermat được Euler mở rộng ra như sau
aφ(n) ≡ 1 (mod n),
Tập 1 Số 3 Tháng 3-2017 Tạp chí Pi 2
với mọi cặp số nguyên (a,n) nguyên tố cùng nhau.
Fermat
Fermat phát biểu định lý nhỏ trong một bức thư gửi cho de Bessy vào năm 1640. Ông nói rằng không viết ra chứng minh vì nó hơi dài. Euler tỏ ra nghi ngờ chứng minh mà Fermat không công bố, nhưng ông rõ ràng rất thích định lý này và công bố ít nhất hai chứng minh khác nhau. Chứng minh đầu tiên dùng công thức nhị thức Newton, còn chứng minh thứ hai dùng tác động của nhóm (ℤ/n)× lên chính nó. Chứng minh thứ hai của Euler thực sự đi trước lịch sử và bản thân chúng ta sẽ phải tiếp tục xuôi dòng lịch sử trước khi quay lại bình luận về chứng minh của Euler.
Euler
3. Disquisitiones của Gauss
Trong chương IX sách “Cơ Sở” của Euclid có ít nhất 5 mệnh đề về cộng và nhân các số chẵn lẻ, ví dụ như tích của hai số lẻ luôn là số lẻ. Chương đầu của quyển Disquisiotiones của
Gauss cũng dành cho việc chứng minh tập ℤ/n các lớp đồng dư modulo n tạo thành một vành. Đáng chú ý rằng, người ta chưa phát biểu một cách tổng quát thế nào là một vành giao hoán vào thời của Gauss, nhưng ông hoàng của Toán học đã biết rõ thế nào là một vành, và hiểu tầm quan trọng của việc ℤ/n tạo thành một vành.
Ông cũng chứng minh rằng vành ℤ/n là một trường khi và chỉ khi n là một số nguyên tố.
Định lý phần dư Trung Hoa có thể được làm mạnh lên như sau:
Nếu với n1, n2, ..., nm là các số nguyên dương đôi một nguyên tố cùng nhau, thì ánh xạ
là đẳng cấu vành.
Gauss
Định lý phần dư Trung Hoa sau này xuất hiện dưới nhiều hình hài khác nhau trong Toán học hiện đại, tường minh nhất là trong đại số, như trong chương Một quyển Cẩm nang Đại số giao hoán của Matsumura, ẩn hơn trong khái niệm adele của Số học hiện đại.
Với mỗi vành giao hoán R, ta ký hiệu R× là tập con các phần tử khả nghịch của R: a ∈ R được gọi là khả nghịch nếu tồn tại b ∈ R sao cho ab = 1. Tập R× là một nhóm giao hoán đối với phép nhân. Sử dụng (1) ta thấy rằng lớp đồng dư của a là một phần tử khả nghịch trong ℤ/n khi và chỉ khi a và n nguyên tố cùng nhau. Như vậy, chỉ số Euler φ(n) chính là số phần tử của nhóm ℤ/n× các phần tử khả nghịch của
Tập 1 Số 3 Tháng 3-2017 Tạp chí Pi 3
ℤ/n. Thể hiện chỉ số Euler như số phần tử khả nghịch của vành ℤ/n cho phép ta giải thích những tính chất cơ bản của chỉ số Euler.
Nếu , với n1, n2, ..., nm đôi một nguyên tố cùng nhau, thì từ (2), ta suy ra rằng ánh xạ
là một đẳng cấu nhóm. Đẳng cấu này kéo theo đẳng thức
đúng nếu các số nguyên dương n1, n2, ..., nm đôi một nguyên tố cùng nhau.
Định lý Fermat nhỏ và mở rộng của nó là định lý Euler cũng có thể suy ra từ cấu trúc nhóm của (ℤ/n)×. Thật vậy, với mọi nhóm hữu hạn G với m phần tử, với mọi g ∈ G, ta có gm = 1. Đây là một hệ quả của định lý Lagrange khẳng định rằng số phần tử của mọi nhóm con H của G luôn là một ước số của số phần tử của G. Áp dụng mệnh đề này vào trường hợp của nhóm con sinh ra bởi một phần tử a ∈ (ℤ/n)×, ta suy ra đẳng thức aφ(n) = 1 trong vành (ℤ/n).
Nhìn từ quan điểm cấu trúc đại số, định lý nhỏ của Fermat và mở rộng của Euler đơn thuần là tính chất đơn giản nhất của nhóm (ℤ/n)× các phần tử khả nghịch của vành (ℤ/n) các lớp đồng dư mod n. Tuy vậy, cần lưu ý rằng Fermat và Euler dường như không ý thức về
điều này.
Nên lưu ý rằng, phải chờ đến thời Galois, Sylvester thì khái niệm về nhóm trừu tượng mới hình thành. Tra cứu chứng minh thứ hai của Eu- ler, ta thấy, tuy rằng ông chưa có khái niệm nhóm trừu tượng nhưng ông đã biết cho nhóm (ℤ/n)× tác động lên chính nó và chứng minh định lý Fermat nhỏ mở rộng dựa vào tác động này. Qua đây, ta thấy rằng khái niệm nhóm tác động lên một tập hợp xuất hiện trước khái niệm nhóm trừu tượng.
4. Định lý nhóm nhân và mật mã
Sylvester đã chỉ ra rằng mọi nhóm Abel hữu hạn đều có dạng
với d1d2··· dm là một dãy số nguyên dương, số sau là bội của số trước. Dãy số d1d2 ···dm, hoàn toàn được xác định bởi A, được gọi là kiểu của A. Số m là số tối thiểu các phần tử lập thành một hệ sinh của A. Nhóm a được gọi là nhóm xích nếu m = 1, nói cách khác, a có một phần tử sinh.
Ta có thể đặt ra câu hỏi xác định kiểu của nhóm nhân (ℤ/n)×. Định lý nhóm nhân khẳng định rằng trong trường hợp n = p là một số nguyên tố thì (ℤ/p)× là nhóm xích.
Giả sử (ℤ/p)× = ℤ/d1 × ℤ/d2 × ··· × ℤ/dm với m > 1 và q là một ước nguyên tố của d1, d2, ... dm. Khi đó, có đúng qm phần tử x ∈ (ℤ/p)× thoả mãn xq = 1. Những phần tử này là nghiệm của
phương trình xq−1 = 0 trong trường Fp = ℤ/p. Vì không thể có quá q nghiệm của phương trình bậc q, ta có m = 1.
Định lý nhóm nhân có nhiều ứng dụng trong lý thuyết số sơ cấp. Ví dụ thứ nhất liên quan đến việc tính toán ký hiệu Legendre.
Cho a và b là hai số nguyên nguyên tố cùng nhau, ta đặt
Xét ví dụ đơn giản nhất với a = −1 và b = p là một số nguyên tố lẻ. Do (ℤ/p)× đẳng cấu với ℤ/(p−1) nên để cho −1 là một bình phương, điều kiện cần và đủ là
Vì vậy, ta có công thức
Tổng quát hơn, Gauss chứng minh luật thuận nghịch bậc hai, một trong những định lý quan trọng nhất trong Disquisitiones. Luật này có thể xem như một thuật toán để tính ký hiệu
Tập 1 Số 3 Tháng 3-2017 Tạp chí Pi 4
Le- gendre. Bên cạnh ý nghĩa thuật toán, nó là khởi nguồn của của một mạch chính trong số học hiện đại, bao gồm luật thuận nghịch Artin, giả thuyết Langlands. Chúng ta sẽ để dành chủ đề rộng lớn này cho những bài khác.
Ứng dụng thứ hai liên quan đến lý thuyết mật mã. Cho n là một số tự nhiên rất lớn và lấy tập hợp ℤ/n làm tập hợp các chữ cái. Mã hoá sẽ là một hàm số ξ: ℤ/n → ℤ/n. Ta giả sử rằng ξ là một song ánh và gọi hàm ngược ξ−1 là giải mã. Nếu n là một số lớn và ξ là một hoán vị bất kỳ, việc xác định ξ−1 có độ phức tạp rất lớn, vượt ra ngoài khả năng tính toán của các máy tính hiện nay.
Xét trường hợp đặc biệt ξ(x) = xa với a là một số nguyên dương lớn hơn 1. Để ánh xạ ξ: ℤ/n → ℤ/n là song ánh, điều kiện cần, nhưng không đủ, là n không có ước chính phương, tức là n = p1 · p2 ··· pm là tích các số nguyên tố
khác nhau. Trong trường hợp này, ta có thể suy ra từ định lý Euler rằng với mọi số nguyên m thoả mãn m ≡ 1 (mod φ(n)) ta có
xm ≡ x (mod n).
Nếu b là một số nguyên dương sao cho ab ≡ 1 (mod φ(n)) thì hàm ξ0: ℤ/n → ℤ/n cho bởi x → xb là hàm ngược của ξ. Như vây để giải mã ξ ta chỉ cần tìm số nguyên dương b sao cho ab ≡ 1
mod φ(n). Với giả thiết a nguyên tố cùng nhau với φ(n), để tìm được b, ta có thể dùng thuật toán chia Euclid. Nói cách khác, để nghịch đảo ánh xạ ξ ta chỉ cần biết chỉ số Euler φ(n).
Từ suy luận này, ta thấy rằng để mã hoá, ta chỉ cần biết n và a; trong khi đó để giải mã, ta cần biết φ(n) và b. Biết φ(n) gần như tương đương với việc phân tích n thành tích các thừa số nguyên tố. Vì phân tích một số nguyên dương lớn thành tích các số nguyên tố có độ
phức tạp tính toán cao cho nên trên nguyên tắc, việc giải mã là rất khó, ngay cả khi việc thông tin về mã hoá có thể hoàn toàn công khai.
5. Tham khảo
Về chủ đề này, bạn đọc có thể tìm đọc thêm cuốn sách kinh điển “Course in Arithmetic” của Serre, hoặc cuốn “A Computational Introduc- tion to Number Theory and Algebra” của Shoup cũng rất tốt (có thể tải về hợp pháp từ trên trang mạng của tác giả). Bạn đọc quan tâm đến lịch sử Toán học nên bắt đầu từ Disquisitiones của Gauss.
Nguồn ảnh: Internet
Tập 1 Số 3 Tháng 3-2017 Tạp chí Pi 5
ỨNG DỤNG TOÁN HỌC TRONG MẬT MÃ VÀ NHỮNG GIẢI PHÁP BẤT NGỜ (Tiếp theo kỳ trước)
Phạm Huy Điển
Phần III. Tiền mật mã - Một sản phẩm của quá trình Toán học hóa mật mã
Tầm quan trọng đặc biệt của chữ ký số, và cũng thể hiện tính hơn hẳn của nó so với chữ ký thông thường, là ở chỗ đảm bảo được sự tin tưởng giữa các bên đối tác, ngay cả khi họ hoàn toàn không có lòng tin vào nhau. Ta đã được thấy điều này trong việc giám sát hiệp ước hạn chế việc thử vũ khí hạt nhân giữa Liên Xô và Mỹ trước đây, và ta sẽ còn thấy điều này được thể hiện rõ nét hơn và phổ biến hơn với sự ra đời của một loại tiền đặc biệt, đó là tiền mật mã.
Sự ra đời của tiền mật mã
Năm 2008, vụ khủng hoảng nhà đất ở Mỹ dẫn đến suy thoái kinh tế toàn cầu, châu Âu cũng rơi vào khủng hoảng vì nợ nần, cục Dự trữ Liên bang Hoa Kỳ (FED) bơm tiền vào cứu trợ nền kinh tế Mỹ. Trong bối cảnh này, lòng tin vào những đồng tiền của chính phủ suy giảm và bắt đầu nảy sinh nghi vấn về những đồng tiền này. Đây chính là thời cơ để cho một loại tiền mới xuất hiện, với tham vọng đáp ứng sự
mong đợi của dân chúng bằng những thuộc tính đặc biệt:
1. Là một loại “tiền trung lập”, không phụ thuộc và không chịu sự can thiệp của bất cứ chính phủ nào, hay một ngân hàng nào trên thế giới;
2. Tiền được giao dịch qua mạng máy tính một cách trực tiếp, ẩn danh và không cần thông qua các ngân hàng (tức là một kiểu tiền mặt dùng trên môi trường mạng, hay có thể gọi là “tiền mặt điện tử”).
Thực ra, nhu cầu về đồng tiền với thuộc tính thứ 2 đã xuất hiện từ khá lâu rồi, do người ta mong muốn có được những giao dịch riêng tư, vượt ra khỏi vòng kiểm soát của các ngân hàng và chính phủ (nhiều người xem đây là một yếu tố cơ bản của quyền con người). Nhu cầu này được xem là chính đáng, một khi Chính phủ có những quyết sách đi ngược lại với xu thế tiến bộ (chẳng hạn như là chính phủ Mỹ đã từng cấm đoán người dân Mỹ ủng hộ Việt Nam trong thời gian xảy ra chiến tranh giữa hai nước). Để có được đồng tiền như thế thì phải có được giải pháp chống gian lận kiểu “chi tiêu 2 lần với 1 đồng tiền” (vì tiền điện tử dễ dàng được “nhân bản”). Đây là vấn đề nan giải mà tất cả các kiểu “tiền kỹ thuật số” ra đời trước năm 2008 đã không thể giải quyết được.
Tập 1 Số 3 Tháng 3-2017 Tạp chí Pi 6
Đồng Bitcoin
Loại tiền mới ra đời không chỉ giải quyết được vấn đề này, mà còn giải quyết hàng hoạt vấn đề nan giải khác, nhờ bản chất của nó được xây dựng trên nền tảng của kỹ thuật mật mã, và do vậy được người ta gọi là tiền mật mã (crypto currency). Đại diện đầu tiên của loại tiền này là đồng Bitcoin. Vì những công cụ quan trọng nhất trong việc tạo ra, lưu trữ và lưu thông (giao dịch) đồng tiền này đều được lấy từ kỹ thuật mật mã, cho nên khẩu hiệu của đồng tiền này là “CHÚNG TA TIN VÀO MẬT MÔ (in cryp- tography we trust), mà không phải là “CHÚNG TA TIN VÀO CHÚA TRỜI” như thấy trên đồng đô la Mỹ.
Đặc tính nổi bật và các thành phần chính của hệ thống Bitcoin
Đặc tính nổi bật nhất của hệ thống Bitcoin là ở chỗ nó vận hành độc lập, không cần sự tham gia hoặc uỷ quyền của bất kỳ ngân hàng hay cơ quan chính phủ nào.
Các thành phần chính của hệ thống này bao gồm:
• Ví người dùng (users’ wallet) là nơi lưu trữ bitcoin, mỗi ví có một cặp chìa khóa riêng. Để nâng cao tính ẩn danh thì mỗi người nên có nhiều ví.
• Chữ ký số (digital signatures) là phương tiện để thực hiện các giao dịch với bitcoin.
• Các giao dịch (transactions): mỗi giao dịch là một chuỗi bit có chứa các thông tin: khóa công khai của ví người chi trả và ví người thụ hưởng, số tiền, và một chữ ký số.
• Xích khối (blockchains) là khái niệm sáng tạo nổi bật của Bitcoin, dùng để thiết lập một trình tự thời gian của các giao dịch và qua đó tránh được việc “chi tiêu 2 lần” (với cùng một đồng tiền). Đây là một cuốn “sổ cái” công cộng liệt kê tất cả các giao dịch, mỗi giao dịch đều được xác nhận hợp lệ bởi một mạng lưới máy tính phân bố phi tập trung. Một khi thông tin về giao dịch đã được ghi vào sổ thì sẽ không bao giờ có thể bị thay đổi hay xóa đi (vì ngoài những giao dịch hiện tại, còn có thông tin liên kết tới khối trước nó, cùng với đáp án của một bài toán rất khó. Đáp án này là khác nhau cho mỗi khối thông tin).
Cơ chế vận hành của Bitcoin
Bitcoin có nền tảng mã nguồn mở và cơ chế mạng ngang hàng
Bitcoin hoạt động dựa vào những thuật toán mật mã cao cấp. Nền tảng kiến trúc của bitcoin có mã nguồn mở (open source), cho phép những ai biết về lập trình đều có thể kiểm tra qua mã nguồn này.
Một mặt, bitcoin là một đơn vị tiền tệ; mặt khác, Bitcoin còn là một mạng lưới chuyển giao tiền tệ phi tập trung, ngang hàng, cho phép người ta có thể gửi bitcoin trực tiếp cho một người khác mà không cần qua một trung gian nào, với một lệ phí cực kì thấp, gần như bằng 0.
Ở đây Bitcoin giải quyết được vấn đề mà từ trước đến nay người ta tưởng như không thể giải được – đó là vấn đề về lòng tin, tức là tạo ra cơ chế để cho người ta tin tưởng lẫn nhau (hay đúng hơn là “không thể lừa nhau”) – bằng việc sử dụng công cụ chữ ký số kết hợp với sáng kiến về xích khối, cùng với các giao thức giao dịch thông minh mang đậm nét mật mã học.
Ai điều hành Bitcoin?
Không một ai hay công ty nào điều hành Bit coin. Nó được vận hành bởi tất cả những người đã đăng ký vào hệ thống dùng Bitcoin, tức là sử dụng phần mềm Bitcoin Clients, hay
Tập 1 Số 3 Tháng 3-2017 Tạp chí Pi 7
còn có một tên gọi thông thường là Wallet (ví tiền điện tử).
Bitcoin được “đào” như thế nào?
Một công cụ then chốt dùng cho xích khối là một hàm băm mật mã H. Cứ mỗi 10 phút thì các giao dịch hiện thời được “đóng gói” và được xác nhận (validated) bởi một thành viên nào đó có được lời giải cho bài toán tìm số nguyên x thỏa mãn phương trình điều kiện sau cho hàm băm mật mã H:
trong đó h là mã băm từ tập các giao dịch trước đó, còn số lượng các số 0 được ấn định trước theo từng thời kỳ (để sao cho một máy tính đủ mạnh vào thời đó có thể tìm được x như thế trong vòng 10 phút). Chính do tính một chiều của hàm băm mật mã mà bài toán trên là vô cùng khó. Thành viên tìm ra x như thế sẽ nhận được phần thưởng bằng các bitcoin (số lượng cũng được ấn định theo từng thời kỳ). Những người tìm x thường được gọi là “thợ mỏ” (như những thợ đào vàng ngày xưa). Việc đào bitcoin là một quá trình thi đua giữa các “thợ đào mỏ”. Vì công nghệ và kĩ thuật càng ngày càng tiến bộ nên các máy tính của các thợ đào cũng càng ngày càng mạnh để đương đầu với độ khó ngày càng gia tăng.
Hiện tại, những “thợ mỏ” chuyên nghiệp nhất đều sử dụng những máy tính song song chuyên dụng, thiết lập trên nền tảng các mạch tích hợp ứng dụng đặc thù ASIC (application
specific integrated circuits). Một số người không dùng ASIC, thường có rất ít khả năng tìm ra được x.
Một ’trang trại máy tính’ dùng cho việc ’đào bitcoin’ tại Iceland.
Đến nay năng lực tính toán của mạng lưới Bit coin đã mạnh hơn gấp 256 lần Top 500 Siêu Máy Tính trên thế giới cộng lại! Như vậy, Bitcoin không phải “từ giấy mà ra” (như tiền của các chính phủ trên thế giới), mà cần phải tốn năng lượng và thời gian để vận hành những cỗ máy tính đào bitcoin, giống như cần phải hao tốn tài nguyên để đào vàng lên từ
lòng đất.
Với đồng bitcoin mọi việc thật đơn giản
Người dùng Bitcoin không cần phải đăng ký tài khoản, không cần nhà bank, không cần thẻ tín dụng, không cần email, không cần biết tên tuổi, địa chỉ, giới tính, quốc tịch,… để nhận hay gửi bitcoin. Số bitcoin người ta có được chỉ
đơn giản nằm trong một hay nhiều cái ví điện tử (địa chỉ mạng) của người đó. Bitcoin trong ví thuộc về người nào đang giữ cái chìa khóa bí mật (nằm trong file wallet.dat), vì chỉ khi có được cái chìa khóa này thì mới có thể “ký” thực hiện giao dịch với bitcoin trong ví.
Người ta có thể biết được số bitcoin đang có trong một cái ví nào đó, nhưng không thể biết được đích danh ai đang sở hữu cái ví đó.
Hiện tại đồng bitcoin có thể được dùng làm gì? Hiện tại bitcoin có thể được dùng để:
• Mua hàng hóa, như xe hơi, phần mềm, vé máy bay,. . . Hiện tại đã có hơn 122000 nhà cung cấp tuyên bố chấp nhận bitcoin làm phương tiện thanh toán (trong đó có những tên tuổi nổi danh như: Microsoft, Dell, WordPress, Newegg, Lamborghini,. . . );
• Chuyển tiền với độ an toàn cao, chi phí thấp, mà không vướng phải rào cản từ phía các ngân hàng.
• Đối phó với các chính sách “nhiễu nhương” của các Chính phủ khi họ gặp khó khăn về tài chính. Chẳng hạn, tháng 3/2013, thế giới tài chính chấn động với sự kiện Nhà nước Cyprus quyết định thu giữ 10% số tiền của những người đã gửi tiền vào nhà bank, khiến dân chúng đổ xô đi mua Bitcoin làm giá tăng vọt. Khi một chính phủ nào đó gây ra lạm phát thì người dân nước đó sẽ mua Bitcoin, để
Tập 1 Số 3 Tháng 3-2017 Tạp chí Pi 8
giao dịch, thay cho đồng tiền đang mất giá, như tình trạng đã từng xảy ra ở Hi Lạp.
• Vì bitcoin trong ví không lưu xuất xứ của nó, nên khi bạn gửi bitcoin cho người khác thì họ có thể biết được số bitcoin đó là từ bạn gửi, nhưng không thể biết được ai là người đã sở hữu số bitcoin đó trước bạn.
Giá trị đích thực của Bitcoin
Nhiều người lầm tưởng rằng giá trị của Bit coin được tính bằng số tiền bạn có thể đổi ra được từ nó, hay nói cách khác, là giá một bitcoin quy ra tiền giấy (USD, Yen, Pounds, Euro, VN Đồng. . . ). Thật sự thì đó không phải là giá trị đích thực của Bitcoin, mà chỉ là một mức giá, một hệ quả có được từ quy luật cung cầu. Giá trị thật sự của Bitcoin nằm ở chính mạng lưới đang vận hành, nơi mà khi người ta tham gia vào thì phải có những đồng xu bitcoin (cũng có thể xem như “cổ phiếu” của hệ thống này). Giá trị của nó nằm trong sự hữu dụng, tiện lợi, an toàn, bảo mật trong việc thanh toán, mua bán (mà không một nhà nước, ngân hàng, hay một công ty nào can thiệp vào được).
Chiếc xe Lamborghini Gallardo (trị giá 210.000 USD) được mua bằng 217 Bitcoin vào tháng 12/2014.
Một số mốc thời gian đáng ghi nhận:
• THÁNG 8, 2008 – Tên miền BIT COIN.ORG được đăng ký.
• THÁNG 10, 2008 – Bản thiết kế Bitcoin được công bố bởi Satoshi Nakamoto.
• THÁNG 10, 2009 – Tỉ giá đầu tiên được công bố bởi New Liberty Standard: 1 USD đổi được 1309.03 Bitcoin.
• THÁNG 11, 2010 – Giá trị của của nền kinh tế Bitcoin vượt mức 1 triệu USD.
• THÁNG 6, 2011 – Giá trị của của nền kinh tế Bitcoin vượt mức $206 TRIỆU USD.
• 2012 – Tạp chí Bitcoin đầu tiên được công bố, lớp học về Bitcoin đầu tiên được dạy tại một trường công, xe hơi đầu tiên được mua bằng bitcoin, bệnh viện đầu tiên nhận bitcoin, website lớn đầu tiên nhận bitcoin.
• 2013 – Ngày 9/4: Một bitcoin đổi được 200 USD; Tháng 10: Máy Bitcoin ATM đầu tiên ra đời tại Vancouver, Canada.
• 2016 – Lượng bitcoin đang lưu thông trên thị trường có giá trị khoảng 6,5 tỷ USD. Tỷ giá đổi tiền công bố trên mạng (XE Currency Converter) là 1 bitcoin bằng 607.65 USD (tại thời điểm 14h21’ ngày 6/10/2016, giờ HN).
• 2017 – Vào lúc 10h sáng ngày 13/2/2017 (thời điểm mà bản thảo này đang được hoàn thành), tỷ giá giữa bitcoin và đồng đô la Mỹ là 1 bitcoin bằng 1002.98 USD.
Những ai có thể bị lừa vì Bitcoin?
Sự phát triển ấn tượng của nền kinh tế Bitcoin đã kéo theo sự ra đời của hàng loạt đồng tiển “ăn theo”, với những chiêu thức quảng cáo khác nhau, và không ít trong số đó đã trở thành công cụ lừa đảo. Khuôn khổ bài viết này không cho phép đề cập đến những đồng tiền đó, nhưng có một điều cần lưu ý là mỗi đồng tiền này thường có một vài đặc điểm riêng khác biệt (mà người ta thường nói là “cải tiến”) so với Bitcoin. Như đã biết, Bitcoin tạo ra cơ chế khiến cho người ta không thể lừa được nhau, nếu như tuân thủ đúng các quy trình giao dịch theo quy định. Do vậy, nếu như ai đó nói rằng mình bị lừa vì Bitcoin thì hoặc là người ấy có sự nhầm lẫn với một đồng tiền khác, hoặc là chưa biết rõ về đồng Bitcoin mà chỉ nghe phong phanh về nó qua sự dẫn dắt của những kẻ lừa đảo, có mưu đồ dẫn dụ người ta tham gia vào mạng lưới đầu tư tiền ảo mang màu sắc “đa cấp”.
Tập 1 Số 3 Tháng 3-2017 Tạp chí Pi 9
Những cản trở và triển vọng
Rõ ràng mục tiêu của Bitcoin là thoát ra khỏi sự kiểm soát của các chính phủ và ngân hàng, cho nên đương nhiên nó không thể nhận được sự ủng hộ của các cơ quan này. Trong thời gian đầu, Bitcoin bị hầu như tất cả các chính phủ và ngân hàng tẩy chay và khuyến cáo người dân nên tránh.
Tuy nhiên, với thời gian, các đặc tính ưu việt của Bitcoin ngày càng trở nên rõ ràng, khiến người ta không thể phủ định được. Trong giới quản lý và các tổ chức tài chính, sự hoài nghi đang dần nhường chỗ cho sự cảm tình, ủng hộ. Đến hôm nay, người ta đã không thể bỏ qua tiềm năng phi thường của xích khối, cùng với các công cụ mật mã khác cho phép những người không tin tưởng nhau có thể làm việc với nhau, mà không cần phải thông qua một cơ quan trung gian nào. Đơn giản là chúng đã làm nên một bộ máy để tạo ra sự tin tưởng.
Ngày càng có nhiều quốc gia thừa nhận Bitcoin (trong đó có cả cộng đồng Châu Âu). Thậm chí có những nước (như Thụy Điển, Singapore,. . . ) đang có kế hoạch phát triển hệ thống tiền tệ kiểu Bitcoin cho đất nước mình. Các ngân hàng lớn trên thế giới cũng đã nhận ra tính ưu việt của công nghệ “xích khối” mà Bitcoin đã sáng tạo ra và mong muốn áp dụng nó. Tóm lại, công nghệ của Bitcoin đã được
khẳng định và đang có xu thế được áp dụng ngày càng rộng rãi trên thế giới.
Cha đẻ của Bitcoin là ai?
Tháng 12/2015 nhà mật mã người Úc là Craig Wright đã tiết lộ thông tin rằng mình là Satoshi Nakamoto, người sinh ra Bitcoin. Năm tháng sau ông đã khẳng định lại, và hiện đang đưa ra các bằng chứng cho điều này.
Sau khi tiếp cận với ông Wright, trang thông tin The Economist cho rằng nhiều khả năng ông ấy chính là Nakamoto, nhưng vẫn còn không ít nghi vấn xung quanh nguồn gốc Bitcoin. vấn đề đang còn phụ thuộc vào những gì ông làm sau khi công khai các thông tin nêu trên. Ông cho biết đã tự gọi mình là Nakamoto theo tên của một triết gia Nhật Bản hồi thế kỷ 17, Tominaga Nakamoto.
Craig Wright. (Nguồn ảnh: Internet)
Tập 1 Số 3 Tháng 3-2017 Tạp chí Pi 10
BÀI TOÁN BRACHISTOCHRONE: Từ lời thách đố đến điều khiển tối ưu Nguyễn Bảo Huy
Lời thách đố những nhà toán học xuất sắc nhất thế giới
Số Tháng Sáu năm 1696 của tờ Acta Eruditorum có đăng một bài toán như sau:
Cho một chất điểm trượt không ma sát dưới tác dụng của trọng lực. Tìm hình dạng của mặt trượt sao cho thời gian vật trượt từ điểm A đến điểm B là nhỏ nhất.
Hình 1: Minh hoạ bài toán brachistochrone.
Thoạt nhìn thì tưởng như đơn giản, nhưng nó không hề tầm thường. Đi kèm bài toán là một lời thách đố như sau (tạm dịch từ bản dịch tiếng Anh, bản gốc là tiếng Latin):
“Tôi, Johann Bernoulli, gửi đến những nhà toán học xuất sắc nhất thế giới. Đối với những con người thông tuệ, không có gì hấp dẫn hơn một bài toán hiểm hóc nan giải, mà lời giải đúng sẽ đem đến cho chủ nhân của nó danh vọng và trường tồn như một tượng đài. Ta đã thấy những ví dụ, như Pascal, Fermat, v.v.. Tôi mong rằng sẽ có được sự ghi nhận của cộng đồng khoa học, khi đặt trước mặt những nhà toán học xuất sắc nhất của thời đại chúng ta một bài toán thử thách các phương pháp và sức mạnh trí tuệ của họ. Nếu ai đó gửi cho tôi lời giải của bài toán này, tôi sẽ tuyên bố công khai, với những lời ngợi ca xứng đáng.”
Cuối cùng, có 5 (năm) nhà toán học đã gửi lời giải cho Johann Bernoulli là:
• Newton, bác này đã quen thuộc với hầu như tất cả mọi người.
• Jacob Bernoulli, anh trai của bác Johann, tác giả của Luật Số lớn trong lý thuyết Xác suất.
• Leibniz, bác này độc lập phát minh phép tính vi phân và tích phân (và tranh chấp cùng với Newton), bác còn là một nhà triết học.
• l’Hôpital, chính là bác đưa ra quy tắc tính giới hạn 0/0 và ∞/∞ quen thuộc.
• Tschirnhaus, bác này là một nhà toán học, nhà vật lý, bác sĩ và nhà triết học tài ba.
Để giải bài toán, Jacob Bernoulli đã phát minh ra một phương pháp mới, mà sau này Euler đã tinh chế lại, dẫn đến sự ra đời của một lĩnh vực toán học quan trọng là phép tính biến phân (cal- culus of variations), và từ đó dẫn tới các ứng dụng trong lý thuyết điều khiển tối ưu (optimal control).
Johann Bernoulli. Nguồn: Internet
Có phải là bài toán tối ưu hoá?
Bài toán brachistochrone đặt ra mục đích đạt được thời gian nhanh nhất, vậy phải chăng nó là một bài toán tối ưu hoá (optimization)? Một bài toán tối ưu hoá được phát biểu như sau:
Cho J là một hàm số của biến x:
J = J(x).
Tập 1 Số 3 Tháng 3-2017 Tạp chí Pi 11
Tìm giá trị của x mà tại đó J(x) đạt giá trị nhỏ nhất (hoặc lớn nhất, thì cũng quy được về bài toán nhỏ nhất).
Hàm số J này được gọi là hàm mục tiêu (cost function). Trong ứng dụng, nó có thể là chi phí sản xuất, thời gian đáp ứng, năng lượng cần thiết, năng lượng tổn thất, v.v. Tối ưu hoá là một lĩnh vực toán ứng dụng được phát triển liên tục cho đến nay.
Bài toán brachistochrone lại không phát biểu thành bài toán tối ưu hoá như vậy được. Vì cái cần tìm là cả một quỹ đạo của chất điểm, chứ không phải chỉ là một giá trị x. Quỹ đạo đó chính là đồ thị của một hàm số của x, ký hiệu là f(x). Như vậy, bài toán được phát biểu như sau:
Tìm hàm số f(x) sao cho:
J = J f(x)
đạt giá trị nhỏ nhất.
Lúc này, J là một “hàm số của hàm số”, được đặt tên mới là phiếm hàm (functional, còn hàm số là function). Việc tìm một hàm số để cực tiểu hoá một phiếm hàm chính là nội dung của phép tính biến phân.
Tại sao lại liên quan đến điều khiển tối ưu? Bài toán điều khiển
Trước khi nói tới điều khiển tối ưu, ta nói một chút về điều khiển. Đối tượng của điều khiển là một hệ có đầu vào và đầu ra; mục đích của điều khiển là tác động tới đầu vào, gọi là luật điều khiển, sao cho đầu ra có chất lượng như mong muốn (xem Hình 3). Ví dụ, khi ta điều chỉnh chân/tay ga để chiếc xe đạt tới tốc độ 60 km/h thì đó chính là điều khiển; đối tượng là chiếc xe, người lái xe là bộ điều khiển.
Hình 3: Cấu trúc một hệ điều khiển.
Làm sao để thực hiện được việc đó một cách tự động (chẳng hạn như xe tự lái đang là trào lưu phát triển hiện nay)? Lúc đó ta cần đến Toán học. Mối quan hệ giữa đầu vào và đầu ra
cần được biểu diễn bởi các phương trình toán học, việc làm này gọi là mô hình hoá (modeling). Đối tượng của điều khiển trong thực tế luôn là các hệ động lực (dynamical system), chúng có mô hình toán học tổng quát như sau:
trong đó:
• u là đầu vào của đối tượng, cũng là biến điều khiển cần tác động.
• x là biến trạng thái, cũng là đầu ra của đối tượng cần được điều khiển. Nói một cách chặt chẽ thì đầu ra là những trạng thái mà ta đo được. Để tránh những tiểu tiết về kỹ thuật, ở đây ta coi trạng thái là đầu ra của hệ.
• t là thời gian. x(t) và u(t) là các hàm số, tức là giá trị của chúng biến đổi theo thời gian.
• d/dt là phép tính đạo hàm (theo biến số là thời gian t). Phương trình có chứa đạo hàm gọi là phương trình vi phân (differen- tial equation). Đạo hàm đặc trưng cho xu thế biến thiên của hàm số (chiều tăng hay giảm, tốc độ biến thiên nhanh hay chậm).
Phương trình này có ý nghĩa rằng: sự biến thiên của trạng thái phụ thuộc vào bản thân trạng thái đó (tức là đặc tính tự thân vốn có của hệ thống) và phụ thuộc vào đầu vào tác động lên hệ thống.
Lấy nguyên hàm ở cả hai vế của phương trình trên, ta sẽ được:
Phương trình này lại có ý nghĩa rằng: trạng thái hiện tại là sự tích luỹ của các trạng thái và đầu vào trong quá khứ của hệ thống, chứ không phải chỉ là kết quả của những tác động đầu vào ở thời điểm hiện tại.
Ta hình dung các tính chất này của hệ động lực qua hình ảnh quen thuộc của Định luật 2
Tập 1 Số 3 Tháng 3-2017 Tạp chí Pi 12
Newton trong Hình 4. Tác động đầu vào là lực F(t), biến trạng thái là vận tốc v(t), đặc trưng động học của hệ là khối lượng M (mang ý nghĩa là quán tính của hệ).
Hình 4: Hệ động lực minh hoạ bằng Định luật 2 Newton.
Sự biến thiên của vận tốc (tức là gia tốc) phụ thuộc vào lực tác động và khối lượng của hệ.
Mặt khác, dù ta có tác động một lực lớn đến đâu đi chăng nữa thì hệ vẫn cần một khoảng thời gian tích luỹ năng lượng để thay đổi vận tốc của nó. Nếu muốn thay đổi vận tốc ngay lập tức thì lực phải lớn vô cùng, đó là điều bất khả thi trong thực tế. Đó là những đặc tính phổ quát của các hệ động lực trong tự nhiên, khiến cho bài toán điều khiển trở nên thú vị.
Đáp ứng đầu ra của hệ chính là nghiệm x(t) của phương trình vi phân nói trên. Thông thường, nếu chỉ có một mình đối tượng thì đáp ứng này không như ta mong muốn. Đó là lý do ta cần thiết kế bộ điều khiển với một lượng đặt mong muốn, tín hiệu phản hồi, và một mô hình toán học của bộ điều khiển đó. Về bản chất, điều đó có nghĩa là ta cần có một hệ
phương trình vi phân mới của cả hệ thống, thay cho mô hình của riêng đối tượng. Hệ phương trình mới này sẽ được tính toán (thiết kế bộ điều khiển) sao cho nghiệm của nó có đặc tính như mong muốn.
Điều khiển tối ưu
Bài toán điều khiển cổ điển đặt ra mục đích đưa hệ thống đến trạng thái mong muốn. Trong khi đó, bài toán điều khiển tối ưu muốn điều khiển trạng thái của hệ sao cho một số chỉ tiêu chất lượng đạt tối ưu (tốt nhất). Bài toán điều khiển tối ưu được phát biểu như sau:
Tìm luật điều khiển u(t) tác động vào hệ thống có mô hình toán học:
dx(t) dt = f x(t),u(t) , sao cho hàm mục tiêu (phản ánh các chỉ tiêu chất lượng cần đạt):
J = J x(t),u(t),
đạt giá trị nhỏ nhất.
Ta thấy:
• J là một hàm số của thời gian t; • J lại là một phiếm hàm của x(t) và u(t).
Cái ta cần tìm không phải là một thời điểm t nào đó mà hàm J đạt giá trị nhỏ nhất. Cái ta cần tìm là cả một luật điều khiển u(t), là một hàm số, khiến cho phiếm hàm J đạt giá trị nhỏ nhất. Bởi vậy, có thể ứng dụng phép biến phân nói ở trên để làm việc này. Điều này đã được nhà toán học Liên Xô Pontryagin thực hiện với nguyên lý cực đại mang tên ông. Đây là một trong hai trụ cột của điều khiển tối ưu cổ điển. Trụ cột còn lại là quy hoạch động (dynamic programming) của Bellman, nhà toán học người Mỹ, cùng thời với Pontryagin. Nếu có dịp ta sẽ nói tiếp về những chuyện này. Quay lại bài toán brachistochrone, hệ có thể được mô hình hoá với vận tốc và vị trí của chất điểm là các biến trạng thái, thời gian di chuyển là phiếm hàm mục tiêu cần tối ưu, và hình dáng của mặt trượt là biến điều khiển. (Các thành phần của trọng lực tác dụng lên chất điểm là hàm của mặt trượt). Ta thấy rằng một bài toán khi phát biểu lên thì không thấy liên quan đến khái niệm “điều khiển”, nhưng lại hoàn toàn có thể giải quyết bằng kỹ thuật điều khiển. Và khi đã được “định dạng” (problem formulation) dưới dạng một bài toán điều khiển tối ưu thì ta có thể giải bài toán đó không chỉ bằng phép biến phân và nguyên lý Pontryagin, mà còn có thể sử dụng rất nhiều kỹ thuật khác của điều khiển tối ưu. Ta tạm khép lại câu chuyện ở đây với một câu nói của John Dewey.
“Một bài toán, hay vấn đề, nếu được đặt ra đúng cách thì đã giải quyết được một nửa”
“A problem well put is half–solved”
JOHN DEWEY
Tập 1 Số 3 Tháng 3-2017 Tạp chí Pi 13
Lời người viết
Các thông tin lịch sử và chuyên môn được tham khảo từ các nguồn tài liệu. Bạn đọc quan tâm có thể dễ dàng tìm kiếm qua những từ khoá đã được cung cấp bằng tiếng Anh. Bài này được viết với mục đích đảm bảo bạn đọc không cần biết về điều khiển và chỉ cần những
kiến thức cơ bản của toán phổ thông là có thể hiểu được. Nếu bạn thấy hay và dễ hiểu thì đó là do sức hấp dẫn của Toán và kỹ thuật điều khiển. Nếu bạn thấy dở và khó hiểu thì đó là do lỗi của tôi.
Tập 1 Số 3 Tháng 3-2017 Tạp chí Pi 14
MỘT SỐ BÀI TOÁN SỐ HỌC - TỔ HỢP
Hà Huy Khoái
Bài viết này nhằm mục tiêu giới thiệu một số bài toán có thể gọi là thuộc loại “số học - tổ hợp”. Thực ra không có một “định nghĩa” nào cho loại bài toán này. Vì thế, bài viết chỉ giới hạn ở việc đưa ra một số ví dụ về một loại bài toán thường gặp trong các kỳ thi học sinh giỏi Toán, mà việc giải chúng đòi hỏi những phương pháp của Số học và Tổ hợp.
Khi trình bày các lời giải, trong chừng mực có thể, chúng tôi cố gắng mô tả quá trình hình thành nên lời giải, hơn là đưa ra một lời giải ngắn gọn.
Để tiện cho việc theo dõi, bài viết được chia thành 5 phần nhỏ: Tỷ số vàng, Dãy nhị phân, Tính chia hết, Biểu diễn số và Trò chơi.
1. Tỷ số vàng
Chúng ta đều biết “tỷ số vàng” sau đây thường xuất hiện trong khoa học, nghệ thuật và đời sống:
Chúng ta cũng thường bắt gặp tỷ số vàng đó trong lời giải của những bài toán số học - tổ hợp. Ta xét bài toán sau.
Bài toán 1.
Giả sử γ,δ là những số vô tỷ dương, thỏa mãn điều kiện
Chứng minh rằng nếu đặt an = [nγ], bn = [nγ] thì mỗi số nguyên dương xuất hiện đúng một lần trong một trong hai dãy an, bn.
Rõ ràng yêu cầu của bài toán tương đương với việc chứng minh rằng, các số nguyên trong mỗi đoạn hữu hạn tùy ý [1; N] có mặt ở một trong hai dãy và mỗi số có mặt đúng một lần. Như vậy, vấn đề chỉ còn là đếm xem trong N −1 số nguyên dương nhỏ hơn N, có bao nhiêu số thuộc một trong hai dãy nói trên.
Tỉ số vàng. Minh họa: Linh Rab.
Xét mọi số nguyên dương n thỏa mãn [nγ] < N, tức là . Dễ thấy, các số n thỏa mãn là
Tập 1 Số 3 Tháng 3-2017 Tạp chí Pi 15
. Tương tự, các số nguyên dương m sao cho .
Như vậy, trong các số nguyên dương nhỏ hơn N, số các số thuộc một trong hai dãy (an),(bn) là Do γ,δ là các số vô tỷ nên Do đó, ta có:
suy ra
Do đó:
Như vậy trong N −1 số nhỏ hơn N có đúng N −1 số thuộc một trong hai dãy đang xét. Từ đây, hiển nhiên ta có điều phải chứng minh.
Bài toán trên đây là một “thành phần” của rất nhiều bài toán tổ hợp. Trong bài viết này, chúng ta sẽ xem xét hai bài toán thuộc loại đó.
Bài toán 2.
Tìm các dãy tăng các số nguyên dương (an), (bm) và thỏa mãn đồng thời các điều kiện sau:
1/ a1 = 1;
2/ Với mọi n ≥ 1,bn = an + n;
3/ an là số nguyên dương nhỏ nhất không thuộc tập hợp {a1,a2,...,an−1,b1,b2,..., bn−1}.
Rõ ràng ba điều kiện nói trên xác định một cách duy nhất các dãy (an),(bn). Hơn nữa, đối với hai dãy tăng, việc thỏa mãn các điều kiện 1/, 2/, 3/ tương đương với việc thỏa mãn các điều kiện 1/, 2/ và 3’/ sau đây:
3’/ Mỗi số nguyên dương đều thuộc một và chỉ một dãy, trong hai dãy đang xét.
Do tính xác định duy nhất của các dãy thỏa mãn 1/, 2/, 3’/, ta có thể tìm ra các dãy đó bằng cách xây dựng cụ thể. Bài toán 1 cho ta cách tìm hai dãy thỏa mãn điều kiện 3’/: đó chính là các dãy an = [nγ], bn = [nδ], trong đó γ, δ là những số vô tỷ dương, thỏa mãn điều kiện
Vấn đề bây giờ chỉ là tìm γ để các điều kiện 1/ và 2/ được thỏa mãn. Để ý rằng
Như vậy, chỉ cần chọn γ vô tỷ, thỏa mãn:
Nghiệm của phương trình trên là “tỷ số vàng” . Như vậy, các dãy (an), (bn) cần tìm là:
Dựa vào phân tích trên đây, độc giả tự trình bày lời giải cho bài toán.
Bài toán 1 và Bài toán 2 lại có thể làm “thành phần” cho bài toán phức tạp hơn sau đây.
Bài toán 3.
Lập dãy số theo cách sau: Lấy x1 = 1, với mỗi i ≥ 2, số xi nhận được từ xi−1 bằng cách đổi (trong biểu diễn thập phân của số xi−1) số 1 thành 01, số 0 thành 1. Làm như vậy, ta nhận được dãy số: 1, 01, 101, 01101,. . . . Trong dãy này, gọi an là vị trí của chữ số 1 thứ n, bn là vị trí của chữ số 0 thứ n (như vậy, a1 = 1, a2 = 3, a3 = 4, b1 = 2, b2 = 5,...). Tìm công thức xác định an và bn.
Tập 1 Số 3 Tháng 3-2017 Tạp chí Pi 16
Phân tích - Lời giải.
Trước tiên, ta cần tìm một công thức xác định mối liên hệ giữa an và bn. Gọi kn là số chữ số 0 đứng trước chữ số 1 thứ n. Theo định nghĩa hai dãy đang xét, ta có
Theo bài ra, chữ số 0 thứ n được “sinh ra” từ chữ số 1 thứ n. Mặt khác, chữ số 1 “biến thành” hai chữ số 01, chữ số 0 biến thành một chữ số 1. Trước chữ số 1 thứ n có kn chữ số 0, và “biến thành” kn chữ số; còn n chữ số 1 “biến thành” 2n chữ số. Từ đó suy ra:
Từ hai công thức trên đây, ta có
Vì an và bn đều là các “số thứ tự” nên hai dãy là dãy tăng, đồng thời mỗi một số nguyên dương xuất hiện đúng một lần trong đúng một trong hai dãy.
Bài toán 1 và Bài toán 2 cho ta đáp số:
Ta sẽ gặp lại “tỷ số vàng” trong phần các bài toán về trò chơi.
2. Các dãy nhị phân
Trong rất nhiều bài toán tổ hợp, đặc biệt là các bài toán “đếm”, ta thường gặp những tình huống mà tại đó có hai khả năng xảy ra; ví dụ như: được tô bởi hai màu, đường đi chỉ được phép sang phải hoặc đi lên, học sinh nam hay nữ, số chẵn hoặc lẻ,. . . Về thực chất, những bài toán như vậy luôn luôn có thể đưa về cùng một dạng phát biểu, trong đó thông thường nhất là dùng các dãy nhị phân (các dãy gồm hai chữ số 0 và 1).
Để hiểu rõ hơn điều vừa nêu, ta xét bài toán sau.
Bài toán 4.
Sau giờ học, các em học sinh xếp hàng để nhận xe đạp ở nhà gửi xe. Giá tiền gửi mỗi xe là 1000 đồng. Giả sử có k em học sinh có tờ 1000 đồng, m em có tờ 2000 đồng. Hỏi có bao nhiêu cách xếp hàng lấy xe sao cho không em nào phải chờ để lấy tiền trả lại? (Với giả thiết người giữ xe không có đồng tiền lẻ nào).
Phân tích - Lời giải
Đây là một bài toán thuộc loại “hai khả năng”: mỗi em học sinh hoặc có tờ 1000 đồng, hoặc có tờ 2000 đồng. Như vậy, để dễ thấy bản chất bài toán, ta có thể lập tương ứng mỗi hàng học sinh với một dãy gồm hai chữ số 0, 1.
Giả sử ứng với mỗi học sinh có tờ 1000 đồng trong hàng, ta viết số 0; ứng với học sinh có tờ 2000 đồng, ta viết số 1. Như vậy, mỗi hàng học sinh tương ứng một dãy gồm k chữ số 0, m chữ số 1. Để tồn tại cách xếp mà không có em nào phải chờ lấy tiền trả lại, điều kiện cần là k ≥ m. Cũng tương tự như trong nhiều bài toán tổ hợp khác, khi việc đếm số phần tử thỏa mãn điều kiện bài ra là khó, ta đếm “phần bù” của nó, tức là đếm những phần tử không thỏa mãn điều kiện bài ra. Bằng cách đó, ở đây ta sẽ xét xem có bao nhiêu hàng mà có học sinh nào đó đến lượt mình phải chờ lấy tiền trả lại.
Theo cách tương ứng của chúng ta, một hàng như vậy sẽ tương ứng một - một với một dãy gồm k số 0, m số 1, trong đó tồn tại vị trí 2s+1 sao cho tại đó có số 1, đồng thời ở 2s vị trí đầu, số số 0 và số số 1 là như nhau. Ta gọi một hàng như vậy là “hàng xấu”.
Nếu bạn là người giữ xe thì bạn sẽ làm thế nào trong tình huống đó? Hiển nhiên là gọi thêm một bạn nào đó có tờ 1000 đồng lên đứng trước hàng! Phương pháp “thực tiễn” này gợi cho ta lập tương ứng mỗi hàng xấu gồm k chữ số 0, m chữ số 1 với một hàng gồm k+1 chữ số
0, m chữ số 1, với chữ số 0 đứng đầu. Đồng thời, trong 2s+2 vị trí đầu tiên, số chữ số 0 và số chữ số 1 là như nhau.
Nếu ta đổi số 0 thành số 1, số 1 thành số 0 ở 2s+2 vị trí đầu, một hàng như trên sẽ tương ứng với một hàng gồm k+1 chữ số 0, m−1 chữ số 1, nhưng với số 1 đứng đầu tiên. Lại bỏ đi
Tập 1 Số 3 Tháng 3-2017 Tạp chí Pi 17
số 1 đầu tiên này, ta được một hàng gồm k+1 chữ số 0, m−1 chữ số 1.
Như vậy, mỗi hàng xấu gồm k chữ số 0, m chữ số 1 sẽ được tương ứng với một hàng gồm k+1 chữ số 0, m−1 chữ số 1 theo cách trên đây.
Dễ thấy rằng, tương ứng như trên là một - một. Thật vậy, xét một hàng tùy ý gồm k+1 chữ số 0, m−1 chữ số 1. Ta thêm số 1 vào đầu hàng, để được hàng gồm k+1 chữ số 0, m chữ
số 1. Do điều kiện k ≥ m nên trong hàng này phải tồn tại vị trí (2s+2 ) mà từ đó trở lên, số số 0 bằng số số 1. Ta đổi số 0 thành số 1 và ngược lại ở các vị trí từ đó trở lên, để được hàng gồm k+1 chữ số 0, m chữ số 1, với số 0 đứng đầu. Bỏ số 0 đầu tiên này, ta nhận được một hàng xấu.
Từ những tương ứng trên suy ra rằng, số các hàng xấu gồm k chữ số 0, m chữ số 1 bằng số các hàng (tùy ý) với k+1 chữ số 0, m−1 chữ số 1, tức là bằng .
Như vậy, số cách xếp hàng sao cho không học sinh nào phải chờ lấy tiền trả lại là:
Bài toán 5.
Có 2k học sinh có chiều cao khác nhau đôi một. Người ta muốn xếp họ thành hai hàng ngang,
sao cho trong mỗi hàng, chiều cao của học sinh giảm dần, và ở mỗi vị trí, em đứng hàng trước cao hơn em đứng hàng sau. Hỏi có bao nhiêu cách xếp hàng thỏa mãn yêu cầu?
Phân tích - Lời giải.
Thoạt nhìn thì đây là bài toán khác hẳn với bài toán trên. Tuy nhiên, nếu ta đã nắm vững nguyên tắc “hai khả năng” thì thấy cả hai bài thuộc cùng một loại. Thật vậy, vấn đề ở đây chỉ là xếp học sinh vào một trong hai hàng.
Với quan niệm như trên, ta cho mỗi em đứng hàng trước cầm một tấm biển ghi số “0”, mỗi em hàng sau cầm tấm biển ghi số “1”.
Sau đó cho tất cả 2k em xếp lại thành một hàng dọc theo thứ tự chiều cao giảm dần. Làm như vậy, ta được một dãy gồm k chữ số 0, k chữ số 1.
Dễ thấy rằng, một cách xếp hàng thỏa mãn điều kiện bài ra chính là một cách xếp hàng sao cho tại mỗi vị trí 2s+1, số số 0 từ vị trí đầu tiên đến đó phải lớn hơn hay bằng s+1.
Theo Bài toán 4, số cách xếp hàng thỏa mãn bài ra là:
(Còn tiếp)
Tập 1 Số 3 Tháng 3-2017 Tạp chí Pi 18
• Mỗi bài toán đề xuất (kèm theo lời giải) cần được nêu rõ là bài sáng tác hay bài sưu tầm (nếu là bài sưu tầm, cần ghi rõ nguồn) và phải được trình bày trong một file riêng đính kèm email (hoặc một tờ giấy riêng).
• Bài giải cho mỗi bài toán cần được trình bày trong một file riêng đính kèm email (hoặc một tờ giấy riêng).
• Người đề xuất bài toán hoặc gửi bài giải cho các bài toán trong Chuyên mục “Thách thức kì này” cần ghi rõ họ, đệm, tên và nơi làm việc / học tập, số điện thoại liên hệ trên trang bài toán, bài giải. Nếu là học sinh (hoặc sinh viên) cần ghi rõ là học sinh lớp mấy (hoặc sinh viên năm thứ mấy).
• Các bài toán đề xuất đã gửi cho Chuyên mục không được công bố hoặc gửi đăng ở bất cứ nơi nào khác, khi chưa có sự đồng ý của Tạp chí Pi.
Thách thức kì này
Dành cho cấp THCS
P31. Cho tam giác nhọn, không cân ABC nội tiếp đường tròn (O). P là một điểm nằm trong tam giác sao cho ∠PAB = ∠PAC. Gọi K là tâm đường tròn ngoại tiếp tam giác PBC. PK cắt trung trực của đoạn thẳng AP tại L. Đường thẳng qua A song song với BC cắt đường tròn (O) tại điểm thứ hai D khác A. Trên đường thẳng AD, lấy E khác A, sao cho LE = LA. Gọi F là điểm đối xứng với A qua OL. Chứng minh rằng đường tròn ngoại tiếp tam giác FED tiếp xúc đường tròn ngoại tiếp tam giác PBC.
Trần Quang Hùng (Hà Nội)
P32. Chứng minh rằng có vô hạn số nguyên dương n có tính chất: n! chia hết cho n3 −1.
Phạm Đức Hiệp (Hà Nội)
P33. Hỏi, trên mặt phẳng, có thể vẽ được nhiều nhất bao nhiêu tam giác mà hai tam giác bất kì (trong số đó) đều có đúng một đỉnh chung và không có điểm nào là đỉnh chung của tất cả các tam giác được vẽ?
Hà Thanh (Cần Thơ)
P34. Cho a,b,c,d là các số thực dương, có tổng bằng 4. Chứng minh rằng
Hỏi đẳng thức xảy ra khi và chỉ khi nào? Nguyễn Văn Quý (Hà Nội)
Dành cho cấp THPT
P35. Tìm tất cả các hàm số f xác định, liên tục trên tập số thực ℝ, lấy giá trị trên ℝ và thỏa mãn điều kiện
Tập 1 Số 3 Tháng 3-2017 Tạp chí Pi 19
với mọi x,y,z,t thuộc ℝ.
Võ Quốc Bá Cẩn (Hà Nội)
P36. Cho số nguyên dương n. Chứng minh rằng tồn tại các số tự nhiên a, b sao cho
Nhóm “TTTH” st.
P37. Cho dãy số thực (xn)n≥1, được xác định như sau: x1 = 1 và
Tính [x2017].
([x] kí hiệu phần nguyên của số thực x).
Nguyễn Duy Liên (Vĩnh Phúc),
Nguyễn Duy Thái Sơn (Đà Nẵng)
P38. Cho tam giác ABC nội tiếp đường tròn (O). Gọi D là hình chiếu vuông góc của A trên BC. Các điểm E,F di động trên (O), sao cho EF ǁ
Giải bài kì trước
P11. Với n là một số nguyên dương, gọi P(n) là tổng lũy thừa bậc n của n số nguyên liên tiếp.
a) Chứng minh rằng P(5) chia hết cho 25.
b) Tìm tất cả các số nguyên dương n > 1 sao cho P(n) chia hết cho n2
Lời giải (dựa theo cách giải của một bạn quên ghi tên).
Với n ∈ N∗ và a ∈ ℤ, đặt
Theo bài ra, ta phải:
a) Chứng minh Pa(5) ⁞ 25 ∀a ∈ ℤ;
b) Tìm tất cả các số nguyên dương n sao cho Pa(n) ⁞ n2∀a ∈ ℤ;
BC. M,N theo thứ tự là hình chiếu vuông góc của B,C trên AE. P,Q theo thứ tự là hình chiếu vuông góc của B,C trên AF. Chứng minh rằng tâm vị tự ngoài và tâm vị tự trong của các đường tròn (DMQ) và (DNP) là các điểm cố định.
Nguyễn Minh Hà (Hà Nội)
P39. Kí hiệu S là tập hợp gồm 2017 số nguyên dương đầu tiên. Hỏi có nhiều nhất bao nhiêu phần tử của S là những số hạng của cùng một cấp số nhân tăng, có các số hạng là số thực và công bội lớn hơn 1?
Hà Huy Khoái (Hà Nội)
P40. Chuẩn bị cho buổi giao lưu của Pi với các bạn học sinh Tiểu học yêu Toán, cô Lê Hương được giao nhiệm vụ chia 500 thanh chocolate (sô-cô-la) giống hệt nhau thành các gói sao cho mỗi gói đều có một số lẻ thanh; cô Cẩm Thơ được giao nhiệm vụ chia 500 cái kẹo mút giống hệt nhau thành các gói sao cho không có 2 gói nào có số kẹo bằng nhau. Kí hiệu s và t tương ứng là số cách chia mà cô Lê Hương và cô Cẩm Thơ có thể thực hiện. Hãy so sánh s và t.
Phan Thị Hà Dương (Hà Nội)
Ta có nhận xét sau:
Nhận xét. Với n là số nguyên dương tùy ý, Pa(n) ≡ Pb(n) (mod n2 ) với mọi a,b ∈ ℤ.
Chứng minh. Với a là số nguyên tùy ý, ta có Suy ra Pa+1(n) ≡ Pa(n) (mod n2).
Từ đó, do a là số nguyên tùy ý nên hiển nhiên ta có điều cần chứng minh.
Trở lại bài toán.
a) Theo Nhận xét, ta có:
Tập 1 Số 3 Tháng 3-2017 Tạp chí Pi 20
b) Xét hai trường hợp sau:
• Trường hợp 1: n là số lẻ.
Đặt n = 2k + 1, k ∈ ℕ. Theo Nhận xét, ta có:
Mà
Nên suy ra Pa(n) ≡ 0 (mod n2 ) ∀a ∈ ℤ. • Trường hợp 2: n là số chẵn.
Đặt n = 2k·m, với k,m ∈ ℕ∗ và m là số lẻ.
Giả sử Khi đó, hiển nhiên ta có . (1)
Mà
và với mỗi j = 0,1,...,m−1, nên từ (1) ta được:
Từ đó, do m lẻ nên
Dễ thấy, ta có k > 1; kéo theo k −1 > 0. Vì thế, Do n chẵn nên
Vì vậy, từ (2) và (3), ta được:
Suy ra
Vì từ (2) ta có (4) nên bằng suy luận “quy nạp lùi” theo k, ta sẽ có:
Điều vô lý vừa nhận được chứng tỏ giả sử ở trên là sai; nghĩa là, với n là số chẵn, không thể có Pa(n) ⁞ n2 với mọi a nguyên.
Vậy, tóm lại, số nguyên dương n thỏa mãn yêu cầu đề bài khi và chỉ khi n là số lẻ.
Chú ý. Trong lời giải trên, có sử dụng một công thức có tên gọi là Công thức nhị thức Newton (Niu-tơn). Theo Chương trình phổ thông môn Toán hiện hành, công thức đó được giảng dạy ở lớp 11. Vì vậy, để các bạn học sinh cấp THCS không gặp khó khăn trong việc theo dõi lời giải, chúng tôi trích dẫn ra đây công thức đó.
Công thức nhị thức Newton: Với n là một số nguyên dương và a,b là các số thực, ta có:
trong đó, với mỗi k = 1,2,..., n−1.
Nhận xét
1. Tất cả các lời giải của bạn đọc mà Tòa soạn nhận được cho tới thời điểm hoàn thiện bản thảo đều cho kết quả đúng ở câu b). Tuy nhiên, ngoài lời giải của bạn quên ghi tên nêu trên, các lời giải còn lại đều không đúng.
2. Có thể giải câu a) một cách mộc mạc như sau (theo tác giả bài toán):
Tập 1 Số 3 Tháng 3-2017 Tạp chí Pi 21
“Xét 5 số nguyên liên tiếp tùy ý: a−2, a−1, a, a+1, a+2 (a ∈ ℤ). Ta có:
Lại có:
Từ hai kết quả nêu trên suy ra P(5) ⁞ 25.”
Có thể thấy, cách giải trên có được nhờ khai thác các tính chất đặc thù của giá trị n = 5. Do đó, không thể áp dụng cách giải này vào việc xử lý câu b) (kể cả khi xét trường hợp n lẻ).
3. Bằng cách dựa vào Nhận xét trong lời giải nêu trên của bài toán và sử dụng định lí Euler (một kết quả mà có thể nhiều bạn học sinh cấp THCS chưa kịp học đến), có thể giải quyết
Từ đó, do m lẻ nên P(n) không chia hết cho 2k+1 ; do đó, P(n) không chia hết cho n 2 vì n2 ⁞ 2k+1.”
4. Trong lời giải nêu trên của bài ra, bằng cách thay đổi một vài chi tiết nhỏ một cách thích hợp, bạn đọc sẽ thu được một chứng minh cho kết quả sau:
“Với n là số nguyên dương, tổng lũy thừa bậc n của n số nguyên liên tiếp chia hết cho n khi và chỉ khi n là số lẻ.”
Lưu Thị Thanh Hà
P12. Cho hai điểm M,N tương ứng nằm trên các cạnh AC,AB của tam giác ABC. Giả sử BM, CN cắt nhau tại P. Biết diện tích các tam giác BPN,BPC,CPM lần lượt bằng 3, 4, 5 (đơn vị diện tích). Hãy tính diện tích tam giác ABC.
Lời giải (của bạn Cao Kỳ Anh, lớp 6C1, Archimedes Academy, Hà Nội).
Nối A với P. Vì tỉ số diện tích của hai tam giác có chung chiều cao bằng tỉ số hai cạnh đáy tương ứng với chiều cao chung đó nên ta có:
trường hợp n chẵn ở câu b) như sau (theo tác
giả bài toán):
“Với n = 2k·m, trong đó k,m ∈ N∗ và m là số lẻ,
xét .
Nhận thấy:
– Với i là số nguyên dương chẵn thì in ⁞ 2n ; suy
ra in ⁞ 22k (do 2n ⁞ 22k).
– Với i là số nguyên dương lẻ thì (i,2k+1 ) = 1.
Do đó, theo định lí Euler, ta có
Suy ra
Từ hai điều trên, với lưu ý từ 1 đến n có n/2 số nguyên dương lẻ (do n chẵn), suy ra
(do SBPN = 3, SBPC = 4 và SCPM = 5 theo giả thiết). Đặt SAPN = 3a (a** là số thực dương), từ (1) ta có SAPC = 4a. Suy ra
và
Tập 1 Số 3 Tháng 3-2017 Tạp chí Pi 22
Từ (2) và (3), ta được Ta có: (5) ⇔ 5(3a+3) = 4(4a−5) ⇔ a = 35.
Thay giá trị a vừa tìm được vào (4), ta được SABC = 252 (đvdt)
Nhận xét. Tất cả các lời giải mà Tòa soạn nhận được tới thời điểm hoàn thiện bản thảo đều đúng và đều tốt. Số lời giải mà Tòa soạn nhận được bằng 1.
Nguyễn Khắc Minh
P13. Tìm tất cả các bộ 3 số hữu tỉ dương a,b,c sao cho đều là các số nguyên.
Lời giải (theo lời giải của bạn Lâm Hữu Phúc, lớp 11 Toán trường PTNK, ĐHQG Tp HCM.)
Đặt với x,y,z,t,m,n là các số nguyên dương và
Khi đó:
Ràng buộc của đề bài tương đương với:
Từ (2) suy ra xz ⁞ y và yt** ⁞ z, zm ⁞ t và nt ⁞ m, mx ⁞ n và yn ⁞ x.
Kết hợp với (1), ta được: z ⁞ y và y ⁞ z, m ⁞ t và t ⁞ m, x ⁞ n và n ⁞ x.
Do đó y = z, t = m, n = x. (3)
Kết hợp với (2), suy ra: (x+t) ⁞ y, (y+x) ⁞ t và (t +y) ⁞ x.
Không mất tổng quát, giả sử x ≥ y ≥ t. Khi đó, ta có: Từ đó, do (t +y) ⁞ x nên chỉ có thể xảy ra 2 trường hợp sau:
• Trường hợp 1:
Khi đó, hiển nhiên ta có x = y = t = z = m = n. Do đó a = b = c = 1.
Bằng cách kiểm tra trực tiếp, ta thấy bộ 3 số (a,b, c) nêu trên thỏa mãn điều kiện đề bài.
• Trường hợp 2:
Khi đó x = t + y. Mà (x+t) ⁞ y (chứng minh trên) nên (y+2t) ⁞ y. Suy ra 2t ⁞ y.
Mà nên chỉ có thể xảy ra 2 trường hợp sau:
+ Trường hợp 2.1:
Lúc này, y = 2t; kéo theo x = 3t. Kết hợp với (3), ta được: z = 2t và n = 3t.
Do đó
Bằng cách kiểm tra trực tiếp, ta thấy bộ 3 số (a,b, c) vừa nêu trên thỏa mãn điều kiện đề bài.
+ Trường hợp 2.2:
Lúc này, y = t; kéo theo x = 2t. Kết hợp với (3), ta được: z = t và n = 2t.
Do đó a = 2, b = 1 và c = 1/2 .
Bằng cách kiểm tra trực tiếp, ta thấy bộ 3 số (a,b,c) vừa nêu trên thỏa mãn điều kiện đề bài.
Dễ thấy, nếu bộ 3 số (a0,b0,c0) thỏa mãn điều kiện đề bài thì mỗi bộ 3 số nhận được từ nó nhờ phép hoán vị vòng quanh cũng là một bộ 3 số thỏa mãn điều kiện đề bài. Vì vậy, từ các kết quả thu được ở trên, ta được:
Tập 1 Số 3 Tháng 3-2017 Tạp chí Pi 23
Có tất cả 7 bộ 3 số hữu tỉ dương thỏa mãn điều kiện đề bài, đó là:
Nhận xét.
1. Cách giải nêu trên là cách giải gần giống với cách giải của tác giả bài toán.
2. Bạn Doãn Quang Tiến (lớp 10A1, trường THPT Phú Mỹ, huyện Tân Thành, tỉnh Bà Rịa - Vũng Tàu) đề xuất một cách giải khác; tóm tắt như sau:
• Từ là các số nguyên suy ra là số nguyên.
• Coi như một phương trình bậc 2 đối với abc, xét tính chính phương của biệt thức, suy ra t = 2 và abc = 1 (sự kiện này cũng đã được chứng minh trong cách giải ở trên bằng một cách khác).
• Đặt chứng minh m,n,p là các số nguyên dương và
• Sử dụng kết quả về nghiệm nguyên dương của phương trình là (3,3,3),(2,4,4),(2,3,6) và các hoán vị để giải quyết bài toán.
Trần Nam Dũng
P14. Có 100 quả bóng đen và 100 quả bóng trắng để trong một chiếc hộp kín. Ta bốc ra ngẫu nhiên 3 quả bóng, và phụ thuộc vào màu của các quả bóng bốc được, ta sẽ bỏ trở lại hộp một số quả bóng theo quy tắc dưới đây:
Bóng bốc ra
Bóng bỏ trở lại
3 quả đen
1 quả đen
2 quả đen, 1 quả trắng
1 quả đen, 1 quả trắng
1 quả đen, 2 quả trắng
2 quả trắng
3 quả trắng
1 quả trắng
Một người đã làm như vậy cho đến khi trong hộp chỉ còn lại 2 quả bóng. Người này không nói cho chúng ta biết quá trình diễn ra như thế nào. Bây giờ ta cần bốc ra một quả bóng từ hộp, nhưng trước đó cần đoán màu của quả
bóng có thể được bốc ra. Hỏi, ta nên đoán quả bóng đó màu gì? Giải thích rõ câu trả lời.
Lời giải (theo lời giải của bạn Nguyễn Minh Uyên, lớp 10T1 trường THPT chuyên Thoại Ngọc Hầu, An Giang).
Ta có các Nhận xét sau:
Nhận xét 1. Theo quy tắc “bốc ra - bỏ lại” của đề bài, trong hộp luôn có ít nhất một quả bóng màu trắng.
Chứng minh. Dễ thấy, ở mỗi lần “bốc ra - bỏ lại”, số quả bóng trắng có trong hộp ngay trước thời điểm bốc ra chỉ bị thay đổi khi việc “bốc ra - bỏ lại” xảy ra theo kiểu thứ tư. Tuy nhiên, trong trường hợp này, luôn có một quả
bóng trắng được bỏ ngược trở lại. Vì thế, trong hộp luôn có ít nhất một quả bóng trắng.
Nhận xét 2. Số quả bóng trắng có trong hộp, sau mỗi lần “bốc ra - bỏ lại”, luôn là một số chẵn.
Chứng minh. Từ quy tắc “bốc ra - bỏ lại” hiển nhiên suy ra số quả bóng trắng có trong hộp sau mỗi lần “bốc ra - bỏ lại” hoặc không đổi hoặc giảm 2. Mà ban đầu trong hộp có một số chẵn quả bóng trắng (100 quả) nên số bóng trắng có trong hộp sau mỗi lần “bốc ra - bỏ lại” luôn là một số chẵn.
Từ hai Nhận xét trên, ta thấy: nếu sau một quá trình thực hiện việc “bốc ra - bỏ lại” mà trong hộp chỉ còn lại 2 quả bóng thì đó phải là 2 quả bóng trắng.
Do đó, nếu cần đoán màu của quả bóng có thể được bốc ra, khi trong hộp chỉ còn lại 2 quả bóng, thì dĩ nhiên ta sẽ đoán đó là quả bóng màu trắng (và khả năng đoán trúng là 100%).
Tập 1 Số 3 Tháng 3-2017 Tạp chí Pi 24
Nhận xét.
1. Thực ra, chỉ cần sau Nhận xét 1 trong lời giải trên, ta cũng đã có thể nói rằng ta sẽ đoán quả bóng có thể được bốc ra khi trong hộp chỉ còn 2 quả bóng là quả bóng màu trắng, do lúc đó xác suất đúng sẽ lớn hơn hay bằng 50%. Tuy nhiên, để khẳng định lựa chọn của chúng ta là tốt nhất, ta cần chứng minh xác suất này thực sự lớn hơn 50% (không nhất thiết phải là 100%). Đó cũng chính là cách tiếp cận của bạn Phan Quốc Vượng (lớp 10 Toán, trường PTNK ĐHQG, Tp HCM), khi giải bài toán. Cụ thể, trong lời giải của mình, sau khi nêu Nhận xét như Nhận xét 1 trong lời giải trên đây, bạn Quốc Vượng đã lập luận như sau:
“Gọi x là xác suất để hai quả bóng còn lại đều màu trắng, y là xác suất để hai quả bóng còn lại gồm một quả màu trắng, một quả màu đen. Vì trường hợp hai quả bóng còn lại đều có màu đen không xảy ra nên x+y = 1.
Trường hợp hai quả bóng còn lại đều có màu trắng có thể xảy ra; chẳng hạn: Ta bốc được ba quả bóng màu đen liên tiếp 49 lần; khi đó, trong hộp sẽ còn 2 quả bóng màu đen và 100 quả bóng màu trắng. Ta bốc được tiếp hai quả bóng màu đen, một quả bóng màu trắng; sau lần bốc này, trong hộp sẽ còn lại 100 quả bóng màu trắng. Cuối cùng, ta bốc được ba quả bóng màu trắng liên tiếp 49 lần; sau 49 lần này, trong hộp còn lại 2 quả bóng màu trắng. Vậy x > 0.
Xác suất bốc được quả bóng màu trắng trong trường hợp hai quả bóng còn lại đều có màu trắng là 1, trong trường hợp còn lại là 1/2. Do đó, xác suất bốc được quả bóng màu trắng là và xác suất bốc được quả bóng màu đen là y/2. Như vậy, xác suất bốc được quả bóng màu đen thấp hơn xác suất bốc được quả bóng màu trắng. Vì vậy, ta nên đoán đó là quả bóng màu trắng.”
2. Bài đã ra thuộc dạng toán về bất biến trong các phép biến đổi trạng thái. Để giải được những bài toán dạng này không cần có những kiến thức cao, cũng không cần có những kỹ thuật biến đổi tốt, mà chỉ cần khả năng nhận xét, phát hiện vấn đề và lập luận tốt.
3. Tính đến thời điểm hoàn thiện bản thảo, Tòa soạn chỉ nhận được hai lời giải đúng cho bài đã ra.
Trần Nam Dũng
P15. Một điểm trên mặt phẳng tọa độ được gọi là điểm nguyên nếu cả hoành độ và tung độ của nó đều là số nguyên. Một điểm nguyên được gọi là điểm nguyên sơ nếu đoạn thẳng nối nó với gốc tọa độ không chứa điểm nguyên nào khác. Cho A là một hình lồi chứa gốc tọa độ O. Đặt tên các điểm nguyên sơ nằm trong A lần lượt theo chiều kim đồng hồ, nhìn từ gốc tọa độ, bởi X1,X2,.... Xét hình lồi B chứa A và có duy nhất điểm nguyên sơ, gọi là Y, nằm giữa X1 và X2, nhìn từ gốc tọa độ. Chứng minh rằng
Lời giải (dựa theo lời giải của bạn Phạm Quang Toàn, học sinh THPT, Queensland, Úc).
Trước hết, ta có Nhận xét sau:
Nhận xét. Không có điểm nguyên nào nằm bên trong hoặc trên cạnh của tam giác OX1X2 trừ các đỉnh của tam giác đó.
Chứng minh. Thật vậy, giả sử ngược lại, có điểm nguyên L, khác O, X1, X2, nằm bên trong hoặc trên cạnh của tam giác OX1X2.
Vì là hình lồi nên A chứa toàn bộ miền tam giác OX1X2 (kể cả các cạnh); do đó, A chứa đoạn thẳng OL. Hơn nữa, do X1,X2 là các điểm nguyên sơ nên tia OL nằm giữa hai tia OX1 và OX2. Vì thế, gọi M là điểm nguyên, khác O, nằm trên đoạn thẳng OL và nằm gần O nhất, ta có M là một điểm nguyên sơ thuộc A và M nằm giữa X1 và X2, nhìn từ O. Điều này mâu thuẫn với cách kí hiệu các điểm nguyên sơ thuộc A. Mâu thuẫn nhận được chứng tỏ giả sử ở trên là sai. Nhận xét được chứng minh.
Ta gọi một đa giác là đa giác nguyên sơ nếu tất cả các đỉnh của nó đều là điểm nguyên và không có điểm nguyên nào nằm trong hay trên cạnh của nó (ngoại trừ các đỉnh).
Theo Nhận xét nêu trên, OX1X2 là một tam giác nguyên sơ.
Do hình lồi B chứa A và do Y là điểm nguyên sơ duy nhất của B nằm giữa X1 và X2, nhìn từ
Tập 1 Số 3 Tháng 3-2017 Tạp chí Pi 25
O, nên bằng cách áp dụng các lập luận đã nêu trong Chứng minh Nhận xét cho hình lồi B và các tam giác OX1Y và OYX2, ta sẽ chứng minh được các tam giác vừa nêu là các tam giác nguyên sơ. Ta thừa nhận Bổ đề sau.
Bổ đề. Diện tích của tam giác nguyên sơ bằng 1/2.
Vì thế,
Từ đó, với lưu ý hai điểm X1,X2 nằm trái phía nhau đối với đường thẳng OY, ta có:
Do Y nằm giữa X1 và X2, nhìn từ O, và Y không nằm trong hay trên cạnh của tam giác OX1X2 (do tam giác này là tam giác nguyên sơ) nên O và Y nằm khác phía nhau đối với đường thẳng
X1X2. Do đó
Từ (1), (2) và (3) suy ra SX1X2Y = 1/2.
Như thế, OX1Y và X1X2Y là hai tam giác có chung cạnh X1Y và có diện tích bằng nhau. Do đó, O và X2 cách đều đường thẳng X1Y. Từ đó, do O và X2 nằm cùng phía đối với đường thẳng X1Y (vì Y nằm giữa X1 và X2, nhìn từ O) nên suy ra OX2 ǁ X1Y.
Chứng minh tương tự, ta sẽ có OX1 ǁ X2Y Vì thế, OX1Y X2 là hình bình hành.
Do đó,
Nhận xét.
1. Bổ đề nêu trong lời giải trên là một trường hợp riêng của định lý Pick về diện tích của đa giác có mọi đỉnh là điểm nguyên (có thể xem phát biểu và một cách chứng minh của định lý Pick trên trang web: https://en.wikipedia.org/wiki/Pick’s_
theorem). Người ta cũng có thể chứng minh được định lý Pick bằng cách dùng chính Bổ đề này. Bạn Phạm Quang Toàn, trong lời giải của mình có đề xuất một chứng minh sơ cấp cho Bổ đề. Tuy nhiên, chứng minh của bạn Toàn không chặt chẽ. Chứng minh chặt chẽ của Bổ
đề sẽ được đề cập trong một bài viết riêng, trên một số tới đây của Tạp chí Pi.
2. Tính tới thời điểm chốt bản thảo, Tòa soạn chỉ nhận được 3 lời giải cho bài toán; trong đó, có một lời giải thiên về cảm tính trong các lập luận (mặc dù đã thừa nhận Bổ đề) và một lời giải sai do hiểu nhầm đề bài (nhầm lẫn giữa vectơ và độ dài đại số).
Nguyễn Duy Thái Sơn
P16. Cho tứ giác ABCD nội tiếp đường tròn (O) và có hai đường chéo AC và BD vuông góc với nhau. Gọi P là giao của AD và BC. Kẻ đường kính PQ của (PCD). Gọi M và N lần lượt là điểm chính giữa cung CD không chứa P và cung CD chứa P của (PCD). Biết QM giao BD,CD lần lượt tại E,F. Còn QN giao AC,CD lần lượt tại K,L. Chứng minh rằng (EDF) tiếp xúc với (KCL).
Lời giải.
Gọi X là giao điểm của AC và BD. Dựng đường tròn đường kính CD đi qua X, cắt đường thẳng MN tại Z, khác phía với X đối với đường thẳng CD. Đường thẳng ZL cắt lại đường tròn đường kính CD tại J.
Do M và N lần lượt là điểm chính giữa các cung CD không chứa P và chứa P của đường tròn PCD, nên MN là trung trực của CD. Do đó ZD = ZC hay Z là điểm chính giữa cung CD của đường tròn đường kính CD. Vì vậy JZ là phân giác của góc vuông DJC. Suy ra .
Hơn nữa, QM và QN tương ứng là phân giác ngoài và phân giác trong kẻ từ Q của tam giác
Tập 1 Số 3 Tháng 3-2017 Tạp chí Pi 26
DQC nên (F,L;D,C) = −1; suy ra J(F,L,D,C) = −1. Từ đó, do JD ⊥ JC nên DJ là phân giác của ∠FJL; suy ra
Gọi Y là giao điểm của KC và MN.
Do MN ⊥ CD nên ta có:
Lại có:
do AC ⊥ BD
Suy ra:
Từ (1) và (2), ta được:
Do đó, LJKC là tứ giác nội tiếp. Chứng minh tương tự, ta cũng được FEDJ là tứ giác nội tiếp. Kẻ tiếp tuyến Jx của (FEDJ). Ta có
nên Jx cũng là tiếp tuyến của (CKL). Suy ra (DEF) và (CKL) tiếp xúc với nhau.
Nhận xét. Tất cả các lời giải Tòa soạn nhận được đều không hoàn chỉnh, do đều bị phụ thuộc vào hình đã vẽ.
Hạ Vũ Anh
P17. Cho tam giác nhọn ABC nội tiếp đường tròn (O). Giả sử OA,OB,OC cắt BC,CA,AB lần lượt tại D,E,F và cắt EF,FD,DE lần lượt tại X,Y,Z. Gọi U,V,W theo thứ tự là hình chiếu vuông góc của X,Y,Z trên BC,CA,AB. Chứng minh rằng AU,BV,CW đồng quy trên đường thẳng Euler của tam giác ABC.
Lời giải (dựa theo các lời giải của bạn Nguyễn Đức Bảo, lớp 11 Toán, trường THPT chuyên Phan Bội Châu, Nghệ An và của một bạn không ghi tên).
Gọi H,G lần lượt là trực tâm và trọng tâm của tam giác ABC. Gọi N là giao điểm của AU và OH. Các đường thẳng AH, AG cắt BC tương ứng tại K, M. Vì G là trọng tâm tam giác ABC nên M là trung điểm của BC và do đó, là hình chiếu vuông góc của O trên BC. Từ tính chất hàng điều hòa cơ bản của tứ giác toàn phần ta có (A,O;X,D) = −1. Chiếu vuông góc hàng điểm điều hòa này lên đường thẳng BC ta được (K,M;U,D) = −1, do đó A(K,M;U,D) = −1.
Chiếu xuyên tâm A chùm điều hòa này lên đường thẳng OH ta được (H,G;N,O) = −1. Từ đó . Như vậy, AU đi qua điểm chia đoạn thẳng HG theo tỷ số −3.
Bằng cách tương tự, ta sẽ cũng sẽ chứng minh được BV,CW đi qua điểm chia đoạn thẳng HG theo tỉ số −3.
Vì vậy, các đường thẳng AU,BV và CW đồng quy tại điểm chia đoạn thẳng HG theo tỉ số −3; do nằm trên HG nên điểm đồng quy này nằm trên đường thẳng Euler của tam giác ABC.
Nhận xét.
Tập 1 Số 3 Tháng 3-2017 Tạp chí Pi 27
1. Các bạn Nguyễn Hoàng Phụng (lớp 12 Toán, trường THPT chuyên Lê Quý Đôn, Ninh Thuận), Trần Vũ Duy (thành phố Hồ Chí Minh), Nguyễn Nguyễn và Trần Minh Nguyên (trường PTNK, ĐHQG Tp.Hồ Chí Minh) cũng có lời giải đúng.
2. Ta có thể coi phép chiếu vuông góc lên đường thẳng BC là phép chiếu song song theo phương OM. Cách nhìn nhận này cho ta một hướng mở rộng, khái quát bài đã ra. Bài toán dưới đây là một trong các bài toán có thể thu được theo hướng đó.
Bài toán khái quát. Cho tam giác ABC có trọng tâm G. Lấy điểm S bất kỳ nằm trong tam giác. Giả sử SA,SB,SC cắt BC,CA,AB lần lượt tại D,E,F và cắt EF,FD,DE lần lượt tại X,Y,Z. Gọi M,N,P tương ứng là trung điểm của các cạnh BC,CA,AB. Lấy U,V,W lần lượt nằm trên các đường thẳng BC,CA,AB sao cho XU ǁ SM,YV ǁ SN,ZW ǁ SP. Chứng minh rằng các đường thẳng AU,BV,CW đồng quy tại một điểm nằm trên đường thẳng SG.
Trần Quang Hùng
P18. Cho P(x) là đa thức bậc n với hệ số thực và cho số thực a ≥ 3. Chứng minh rằng ta có bất đẳng thức
Đặt
Ta có Q(x) là đa thức bậc n−1, với hệ số thực. Do đó, theo giả thiết quy nạp, tồn tại số nguyên k, với 0 ≤ k ≤ n, sao cho
Ta có
Mà
nên
Lời giải (của bạn Nguyễn Minh Hải, lớp 11, THPT chuyên Quốc học, Huế).
Ta chứng minh bài toán bằng phương pháp quy nạp theo n.
Với n = 0, xét đa thức hằng P(x) ≡ c. Ta có Như vậy, kết luận của bài toán đúng với n = 0.
Giả sử kết luận của bài toán đã đúng cho các đa thức bậc n−1, với hệ số thực (n ≥ 1).
Xét đa thức P(x) tùy ý bậc n, với hệ số thực.
Suy ra
Do đó,
Như vậy, kết luận của bài toán đúng cho các đa thức bậc n, với hệ số thực.
Theo nguyên lý quy nạp, ta có điều phải chứng minh.
Nhận xét.
1. Có thể thấy, lời giải trên, về thực chất, đã chứng minh được một kết quả mạnh hơn kết luận của bài ra: Với các giả thiết như ở bài đã ra, ta có bất đẳng thức
Tập 1 Số 3 Tháng 3-2017 Tạp chí Pi 28
2. Bạn Nguyễn Nguyễn (lớp 10 Toán, trường PTNK, ĐHQG Tp. Hồ Chí Minh) có lời giải tương đối tốt. Bạn Nguyễn đã sử dụng công thức nội suy Lagrange để giải bài toán và cũng đi đến kết luận mạnh hơn nói trên.
Nguyễn Duy Thái Sơn
P19. Cho p là một số nguyên tố thỏa mãn p ≡ 11 (mod 12). Đặt M = {1,2,..., p−1} và lấy S là một tập con bất kì của M có phần tử. Ta kí hiệu ∆s là thặng dư không âm bé nhất của theo modulo p, tức là
Đặt . Chứng minh rằng
Lời giải (dựa theo lời giải của bạn Phạm Quang Toàn, học sinh THPT, Queensland, Úc).
Ta sẽ chứng minh kết luận của bài toán với giả thiết p là số nguyên tố tùy ý lớn hơn 2. Trước hết, ta chứng minh Bổ đề sau.
Bổ đề. Cho số nguyên tố p. Khi đó, với mỗi số nguyên dương a ∈ M, luôn tồn tại duy nhất số nguyên b ∈ M sao cho ab ≡ 1 (mod p). Ta viết b = a−1 .
Chứng minh. Với mỗi a ∈ M, xét tập X = {0,a,2a,...,(p−1)a}. Vì với mọi 0 ≤ i < j ≤ p − 1 nên X là một hệ thặng dư đầy đủ modulo p. Suy ra, tồn tại duy nhất số b ∈ M sao cho ab ≡ 1 (mod p).
Trở lại bài toán.
Theo Định lý Wilson, ta có:
Do đó,
Suy ra
Tiếp theo, ta chứng minh với mỗi a ∈ M, tồn tại tập S ⊂ M sao cho (mod p); nghĩa là, tồn tai tập S thỏa mãn điều kiện đề bài sao cho a + a−1 = ∆s (mod p). (2)
Thật vậy, trước hết, nhận thấy có tất cả cặp (a,a−1 ), a ∈ M, đôi một rời nhau, vì:
– Với mỗi 1 < a < p−1 tồn tại duy nhất số b ≠ a,b ∈ M, sao cho a−1 = b (theo Bổ đề),
– Với a = 1 hoặc a = p−1 thì a−1 = a.
Xét a tùy ý thuộc M. Xét hai trường hợp sau: • Trường hợp 1. p ≡ 1 (mod 4).
– Nếu a = 1, xét tất cả các cặp (x, x−1 ), x ∈ M {1, p−1}. Có tất cả cặp như vậy. Lấy S là tập gồm cặp tùy ý trong số đó, ta được tập S thỏa mãn yêu cầu.
– Nếu a ≠ 1, xét tất cả các cặp (x, x−1 ), x ∈ M {1, p−1,a,a−1}. Có tất cả cặp như vậy. Lấy S là tập gồm 1, a và cặp tùy ý trong số đó, ta được tập S thỏa mãn yêu cầu.
• Trường hợp 2. p ≡ 3 (mod 4).
Xét tất cả các cặp (x, x−1), x ∈ M {1, p - 1, a, a−1}. Có tất cả cặp như vậy.
Lấy S là tập gồm **a và cặp tùy ý trong số đó, ta được tập S thỏa mãn yêu cầu.
(2) được chứng minh.
Từ (1) và (2), hiển nhiên suy ra A = T. Để ý rằng, với a,b ∈ M,a < b, ta có:
Tập 1 Số 3 Tháng 3-2017 Tạp chí Pi 29
Do đó, . Vì vậy,
Nhận xét. Trong số các lời giải Tòa soạn nhận được tới thời điểm hoàn thiện bản thảo, có 4 lời giải đúng. Tất cả các bạn có lời giải đúng đều nhận xét điều kiện p ≡ 11 (mod 12) là hơi chặt. Các bạn Trần Minh Nguyên và Nguyễn Nguyễn (học sinh trường PTNK, ĐHQG Tp. Hồ Chí Minh) đưa ra lời giải đúng cho bài toán, với giả thiết p ≡ 3 (mod 4). Một bạn không ghi tên cũng giải đúng bài toán trong trường hợp p ≡ 3 (mod 4) và nhận xét có thể làm được tương tự như vậy khi p ≡ 1 (mod 4).
Lê Anh Vinh
P20. Trong mặt phẳng, xét 7 điểm đôi một phân biệt. Người ta muốn vẽ các đường tròn đi qua đúng 4 trong 7 điểm này. Hỏi có thể vẽ được nhiều nhất bao nhiêu đường tròn?
Lời giải (dựa theo lời giải của tác giả bài toán).
Kí hiệu 7 điểm đang xét là Ai,i = 1,7.
Giả sử vẽ được n đường tròn, n ∈ N∗, thỏa mãn điều kiện đề bài.
Với mỗi i = 1,7, kí hiệu si là số đường tròn đi qua điểm Ai.
Do mỗi đường tròn đi qua đúng 4 điểm nên . Với mỗi i = 1,7, gọi mi là số cặp đường tròn mà hai đường tròn trong mỗi cặp cùng đi qua điểm Ai. Đặt , ta có:
Mặt khác, do 2 đường tròn tùy ý (trong n đường tròn vẽ được) cắt nhau ở tối đa 2 điểm trong số 7 điểm Ai nên
Từ (1) và (2), suy ra
Ta có: (3) ⇔ n ≤ 7.
Xét n = 7. Khi đó, ở tất cả các đánh giá nêu trên phải xảy ra dấu “=”. Vì thế, 7 đường tròn vẽ được phải thỏa mãn đồng thời các điều kiện sau:
1/ Mỗi đường tròn đi qua đúng 4 điểm và qua mỗi điểm có đúng 4 đường tròn;
2/ Hai đường tròn tùy ý cắt nhau ở đúng 2 điểm trong 7 điểm đang xét.
Xét một điểm tùy ý trong 7 điểm, giả sử là A1. Vì 2/ nên phép nghịch đảo P tâm A1, tỉ số k ≠ 0, sẽ biến 4 đường tròn đi qua A1 thành 4 đường thẳng không đi qua A1 và đôi một cắt nhau, tạo thành một tứ giác toàn phần như Hình 1 dưới đây.
Vì mỗi đường tròn trong 3 đường tròn không đi qua A1, đều đi qua đúng 4 điểm trong số 6 điểm khác A1 (theo 1/) và đều cắt từng đường tròn trong 6 đường tròn còn lại tại 2 điểm trong 6 điểm đó (theo 2/) nên phép nghịch đảo P biến 3 đường tròn không đi qua A1 thành 3 đường tròn phân biệt, mỗi đường tròn đi qua đúng 4 điểm trong 6 điểm A,B,C,D,E,F ở hình trên. Tuy nhiên, dễ thấy, chỉ có thể có tối đa 2 đường tròn phân biệt, mỗi đường tròn đi qua 4 điểm trong 6 điểm vừa
Tập 1 Số 3 Tháng 3-2017 Tạp chí Pi 30
nêu. Mâu thuẫn nhận được chứng tỏ n ≠ 7. Do đó, n ≤ 6.
Hình 1
Xét 7 điểm gồm 3 đỉnh của tam giác nhọn ABC, trực tâm H của tam giác đó và 3 chân đường cao D,E,F hạ từ A,B,C xuống các cạnh BC,CA,AB (xem Hình 2 dưới đây).
Hình 2
Dễ thấy, 6 đường tròn (AFHE),(BFHD),(CDHE), (ABDE),(BFEC) và (AFDC) thỏa mãn các điều kiện của đề bài. Vậy, chỉ có thể vẽ được nhiều nhất 6 đường tròn thỏa mãn các yêu cầu của đề bài.
Nhận xét. Mặc dù thời hạn gửi lời giải cho bài toán đã được “âm thầm” kéo dài thêm 1 tháng, tới thời điểm số Tạp chí này lên trang, Tòa soạn vẫn không nhận được một lời giải nào gửi đến, từ bạn đọc.
Nguyễn Khắc Minh
Tập 1 Số 3 Tháng 3-2017 Tạp chí Pi 31
Nguyễn Thanh Giang1
Trong lịch sử loài người, có một con số đã làm say mê rất nhiều người. Qua nhiều thế hệ, từ cổ xưa đến hiện đại, nhiều bộ óc phi thường đã tìm cách tính con số đó - đó chính là số Pi. Dường như ta có thể gặp Pi không chỉ trong Toán học, mà Pi hiện diện khắp nơi.
1 Phó Hiệu trưởng Trường THPT chuyên Hưng Yên – một người thày tôi đã từng được học (toankho.com)
Có thể cho rằng, Pi điều khiển hành trình trôi xuôi của những dòng sông uốn khúc, từ Amazon cho tới sông Thames. Độ uốn khúc của một con sông được mô tả bằng tính ngoằn ngoèo của nó - chiều dài (tính dọc theo chiều dài uốn khúc) chia cho khoảng cách từ nguồn nước đến đại dương tính theo đường chim bay. Hóa ra con sông trung bình thì có độ uốn khúc khoảng chừng 3,14, ba chữ số đầu của số Pi.
Trong quyển sách “Những cuộc phiêu lưu của Alex vào Miền đất số”, Alex Bellos mô tả số Pi đã truyền cảm hứng cho một dạng kĩ xảo đặc biệt của tác phẩm sáng tạo gọi là phong cách Pil- ish. Đây là những bài thơ, trong đó số kí tự của những từ liên tiếp tuân theo các chữ số
của Pi. Một trong những bài thơ thành công nhất là bài Cadaeic Cadenza của Mike Keith. Nó bắt đầu với những dòng: One/ A poem/ A raven (Một/ Một bài thơ/ Một con quạ), One có 3 kí tự, A có một ký tự, poem có 4 kí tự, A có một kí tự, raven có 5 kí tự, tương ứng với 3,1415 và tiếp tục như vậy cho đến 3835 chữ số còn lại! Tác phẩm “Not A Wake” của Mike Keith, quyển sách đầu tiên viết hoàn toàn theo phong cách Pilish được bắt đầu như sau:
Now I fall, a tired suburbian in liquid under the trees
Drifting alongside forests simmering red in the twilight over Europe
Now có 3 kí tự, I có 1 kí tự, fall có 4 kí tự, a có 1 kí tự,. . . Ngoài ra Keith còn viết một quyển sách 10.000 từ sử dụng kĩ thuật trên.
Để dễ nhớ các chữ số sau dấu phẩy của Pi, người ta đã tìm ra nhiều cách, trong đó có các câu bằng chữ. Ở Việt Nam có câu tiếng Việt của nhà giáo Vũ Hữu Bình: “Một ý nghĩ ý tưởng trong sáng là phương hướng cho nhiều hành động cao thượng” (Tiếng Việt không có từ nhiều hơn 7 chữ cái nên phải dùng hai từ có dấu - để chỉ các số 8,9 chữ cái). Đây là câu cho giá trị gần đúng của Pi với 12 chữ số sau dấu phẩy 3,141 592 653 589.
Theo sách kỷ lục thế giới Guiness chàng sinh viên có trí nhớ siêu đẳng Poh Ann người Malaixia khi 21 tuổi chỉ với 15 giờ vừa đọc, vừa ghi lại được 67053 chữ số sau dấu phẩy của số Pi phá kỷ lục 59956 chữ số trước đó của một người Malaixia khác.
Tập 1 Số 3 Tháng 3-2017 Tạp chí Pi 32
Những chiếc bánh được trang trí với kí tự Pi trong ngày 14.3
Sự hiện diện kỳ diệu của số Pi từ vũ trụ tới địa lý, những câu chuyện thú vị của Pi từ văn chương đến cuộc sống là những khám phá lý thú, không ngừng nghỉ của những người yêu toán học trên toàn thế giới. Đó cũng là lý do, ngày 14.3 (số Pi với 2 số lẻ đầu tiên 3,14) được gọi là Ngày số Pi. Ngày số Pi được tổ chức lần đầu tiên ở San Francisco Exploratorium (Mỹ) vào năm 1988, theo ý tưởng của Larry Shaw. Ngày số Pi gần đúng là ngày 22.7, bởi mọi người vẫn biểu diễn Pi là 22/7 (hai mươi hai phần bẩy).
Albert Einstein cũng sinh vào ngày Pi, 14.3. Số Pi đã được biết tới gần 4.000 năm trước bởi người Babylon cổ đại. Một bản khắc có niên đại từ năm 1900 đến 1680 trước Công nguyên tìm thấy Pi là 3,125. Người Ai Cập cổ đại có những khám phá tương tự, như bằng chứng nêu trong sách giấy cói Rhind hồi năm 1650 trước Công nguyên. Trong văn tự này, người Ai Cập đã tính diện tích của một vòng tròn bằng một công thức cho số Pi một giá trị gần đúng là 3,1605. Cả trong Kinh thánh (bản King James) cũng có nói Pi được tính gần đúng.
Tính toán đầu tiên của số Pi được thực hiện bởi Archimedes xứ Syracuse (287 - 212 trước CN). Archimedes đã sử dụng định lí Pythagoras để tính diện tích của hai đa giác. Archimedes lấy gần đúng diện tích của một vòng tròn dựa trên diện tích của một đa giác đều nội tiếp bên trong vòng tròn và diện tích của một đa giác đều ngoại tiếp vòng tròn đó.
Ở Trung Quốc, vào thời Ngụy Tấn (khoảng năm 263), nhà toán học Lưu Huy đã chỉ ra được giá trị của số Pi là 3,1416. Đến thời Nam
- Bắc Triều (khoảng năm 480), nhà khoa học Tổ Xung Chi đã tìm ra số 355/113, hay giá trị của số Pi nằm trong khoảng từ 3,1415926 đến 3,1415927. Đây là số Pi được xác định chính xác nhất trong vòng 900 năm sau đó.
Nhà khoa học Tổ Xung Chi. Nguồn: Internet
Khoảng năm 1450, Al’Khashi tính toán số Pi với 14 chữ số nhờ phương pháp đa giác của Archimedes. Đó là lần đầu tiên trong lịch sử nhân loại đã tìm được con số Pi với trên 10 chữ số.
Năm 1609 Ludolph von Ceulen, nhờ phương pháp của Archimedes, đã tính được số Pi với 34 chữ số mà người ta đã khắc số này trên mộ bia của ông.
Pi bắt đầu được kí hiệu bằng kí tự π vào năm 1706 bởi nhà toán học người Anh William Jones. Jones dùng số 3,14159 cho Pi, mặc dù làm thế nào ông tính được con số này thì vẫn là bí ẩn, vì tác phẩm của ông đã bị thất lạc.
Nhà toán học Archimedes. Nguồn: Internet
Tập 1 Số 3 Tháng 3-2017 Tạp chí Pi 33
Phải đợi đến thế kỷ thứ 18 và đầu thế kỷ thứ 20 thì số Pi đã được tính với độ chính xác là 1000 chữ số sau dấu phẩy. Năm 1995, Hyroyuki Gotu đã chiếm kỷ lục thế giới: tìm ra 42195 chữ số sau dấu phẩy. Cuối thế kỷ 20, người ta đã tính được Pi với độ chính xác đến con số thứ 200 tỉ. Năm 2002, một nhà toán học người Nhật Bản đã sử dụng siêu máy tính để tính toán chính xác số Pi đến chữ số thứ 1.241.100.000.000 (một nghìn hai trăm bốn mươi mốt tỉ một trăm triệu chữ số).
Tháng 8 - 2009, con số thứ 2,6 tỉ tỉ của Pi được xác định bởi nhà khoa học Daisuke Takahashi tại Đại học Tsukuba, Nhật Bản.
Cuối năm 2009 nhà khoa học máy tính người Pháp là Fabrice Bellard đã phá kỷ lục của Daisuke Takahashi khi tính toán chính xác đến con số thứ 2,7 tỉ tỉ của Pi. Mất đến 131 ngày để tính toán, nhưng đây là một kết quả cực kỳ ấn tượng vì Fabrice Bellard chỉ dùng máy tính để bàn thông thường xử lý số liệu. Đạt được kết quả trên là nhờ Fabrice Bellard đã phát triển một phần mềm xử lý thuật toán mạnh hơn 20 lần so với những sản phẩm tương tự trước đó.
Ngày nay nhiều nhà khoa học máy tính, nhiều nhà toán học vẫn tiếp tục tính toán các chữ số tiếp theo của số Pi và nghiên cứu quy luật các chữ số đó để tìm kiếm sự kỳ diệu của Pi.
Chú thích của tác giả: Bài viết có sử dụng thông tin mạng từ Wikipedia, website Pi - Search Page, The Next web, báo Thanh niên và cuốn sách “Các câu chuyện toán học - Tập 1: Tất nhiên và ngẫu nhiên” của Nguyễn Bá Đô và Nguyễn Hồng Minh (NXB Giáo dục 2001).
Chú thích của Tạp chí Pi:
- Số Pi dường như theo suốt cuộc đời mỗi con người: từ lúc học tính diện tích hình tròn, đến khi giải quyết những vấn đề khó khăn của công nghệ và kỹ thuật, hay trong nghiên cứu Toán học, dù là ở bất kỳ chuyên ngành nào. Bởi thế nên khi ra một tạp chí Toán học hướng đến cộng đồng, cái tên được chọn gần như là duy nhất thích hợp: Tạp chí Pi.
- Cuộc thi “Tôi và Toán học” của Tạp chí Pi quy định thời gian nhận bài từ 14/3 đến 22/7.
- Có một chuyện vui về số Pi ở Việt Nam. Khoảng năm 1969 - 1970, ông Trí Uẩn, người giới thiệu và quảng bá trò chơi “Bảy miếng xếp ngàn hình” nổi tiếng, gửi đến Ban Toán – Lý, Uỷ ban Khoa học và Kỹ thuật Nhà nước một công trình tính số Pi, mà ông gọi là số U. Ông dùng phương pháp cắt ghép như trong trò chơi Trí Uẩn, để dựng một hình vuông có diện tích bằng diện tích một hình tròn cho trước, từ đó đưa ra giá trị của Pi, một số hữu tỷ. Theo ông đó mới là giá trị chính xác. Cách tính này được ông diễn ca thành bài thơ lục bát rất hay, đến nỗi TBT Pi đến bây giờ vẫn còn nhớ mấy câu:
Ác-si-mét chết vẫn còn
Đá cầu, đá trụ hai hòn trơ trơ
Trên mồ để lại ước mơ
Hình tròn vuông đó cậy nhờ mai sau...
Ý ông Trí Uẩn là nếu Acsimet mà biết giá trị chính xác của số Pi (số U) thì đã không ôm hận vì bài toán cầu phương hình tròn, đã không di chúc cho hậu thế đặt hai hòn “đá cầu, đá trụ” trên mồ ông. Tất nhiên về mặt Toán học thì ông Trí Uẩn chứng minh không chính xác, vì Pi là số siêu việt, nhưng phải thừa nhận ông cắt ghép hình rất tài tình, và tìm ra giá trị gần đúng của số Pi.
Tập 1 Số 3 Tháng 3-2017 Tạp chí Pi 34
Lê Văn Thiêm con người và sự nghiệp
Hà Huy Khoái
I. Sơ lược tiểu sử
Lê Văn Thiêm sinh ngày 29 tháng 3 năm 1918 tại làng Trung Lễ, Đức Thọ, Hà Tĩnh. Trung Lễ là một làng cổ, thành lập cách đây khoảng 600 năm trên vùng đất trũng, quanh năm bị đe doạ vì nạn hạn hán, lụt lội. Họ Lê ở Trung Lễ
nổi tiếng về truyền thống Nho học và yêu nước. Thân sinh ra Lê Văn Thiêm là cụ Lê Văn Nhiễu (1869-1929), đậu cử nhân Khoa Canh Tý (1900). Mẫu thân của cụ Cử Lê Văn Nhiễu, tức bà nội của Lê Văn Thiêm, là cụ bà Phan Thị Đại, chị ruột nhà yêu nước Phan Đình Phùng. Chú ruột của Lê Văn Thiêm là ông Lê Văn Huân, đậu Giải nguyên Khoa Bính ngọ (1906), tham gia phong trào yêu nước Duy Tân hội, rồi Tân Việt Đảng, và tự sát trong nhà lao Vinh năm 1929.
Cụ Lê Văn Nhiễu tuy đỗ đạt nhưng không ra làm quan, mà ở lại quê nhà dạy học, bốc thuốc, phụng dưỡng cha mẹ, nuôi dạy con cái. Cụ sinh được 13 người con, 8 người con trai, 5 người con gái.
Người anh cả của Lê Văn Thiêm là Lê Văn Kỷ đậu Tiến sĩ năm Kỷ Mùi (1919) trong khoa thi cuối cùng của Triều Nguyễn. Vậy là cụ Cử Nhiễu có một người con đậu Tiến sĩ cuối cùng
của nền Hán học, và một người con đậu Tiến sĩ đầu tiên của nền Tây học nước nhà! Anh thứ hai của Lê Văn Thiêm, ông Lê Văn Luân, là Bí
thư Huyện uỷ Đảng Cộng sản Đông Dương Huyện Đức Thọ, bị Pháp xử tử hình năm 1931. Trong số 5 người chị gái của Lê Văn Thiêm có hai người tham gia phong trào cách mạng 1930-1931 và được công nhận là Lão thành cách mạng.
Cử nhân Lê Văn Nhiễu thân sinh của GS Lê Văn Thiêm
Lê Văn Thiêm là con út trong nhà, nên khi còn bé, được đặt tên là “Thêm”, tức là đứa con “Trời cho thêm”. Khi ra đời, cậu bé Thêm rất yếu, vì bà mẹ đã sinh nở đến lần thứ 13. Mẹ cậu không còn sữa, nên cậu phải bú nhờ người
Tập 1 Số 3 Tháng 3-2017 Tạp chí Pi 35
chị dâu tên là Sâm, vợ của anh Lê Văn Luân. Vì thế, đối với cậu, bà Sâm cũng gần như người mẹ thứ hai. Ông Luân, bà Sâm đều hoạt động cho Tân Việt Đảng. Bà đóng vai người bán hàng tơ lụa, ông đóng vai người chở thuê, hai người đi khắp nơi tuyên truyền cách mạng, in tài liệu, rải truyền đơn. Khi còn nhỏ, cậu bé Thêm học ở quê nhà với chú ruột, Giải nguyên Lê Văn Huân. Cậu nổi tiếng học giỏi, nhưng cũng nổi tiếng là “khờ” . Lớn lên, Lê Văn Thiêm theo anh cả - ông Nghè Kỷ, đi học ở Huế, rồi ở Quy Nhơn.
Sinh ra trong một gia đình giàu truyền thống yêu nước, anh thanh niên Lê Văn Thiêm sớm nuôi trong mình hoài bão học tập để phụng sự Tổ quốc. Năm 1941, Lê Văn Thiêm thi đỗ vào trường Ecole Normale Supérieure ở Phố
d’Ulm của Paris. Đó là trường đại học danh giá nhất nước Pháp, nơi đào tạo những nhà khoa học nổi tiếng nhất. Thi đỗ vào Ecole Normale là một vinh dự lớn đối với bất kỳ học sinh nào của nước Pháp. Tốt nghiệp Ecole Normale, Lê Văn Thiêm tiếp tục làm luận án Tiến sĩ tại Đức, rồi luận án Tiến sĩ quốc gia tại Pháp. Ông đã từng học với những người thầy giỏi nhất thời đó, như Nevan- linna, Teichmuler, Valiron, và nghiên cứu một lĩnh vực thời sự nhất thời bấy giờ: lý thuyết phân phối giá trị các hàm phân hình. Ông bảo vệ luận án Tiến sĩ quốc gia năm 1949 với những kết quả mà ngày nay đã trở thành kinh điển.
Nhờ những kết quả xuất sắc trong nghiên cứu khoa học, năm 1949, Lê Văn Thiêm nhận được một ghế giáo sư tại trường Đại học Zurich, Thuỵ Sĩ. Ông là người Việt Nam đầu tiên nhận chức giáo sư ở một đại học danh tiếng của Châu Âu. Một chỗ làm việc tuỵêt vời, một hướng nghiên cứu thời sự, những kết quả đầu tay đã trở thành nổi tiếng, tất cả đều mở ra trước mắt nhà toán học trẻ Lê Văn Thiêm một con đường thênh thang để đi đến những đỉnh cao của khoa học.
Nhưng mục đích của đời ông trước hết là đóng góp sức mình cho cuộc đấu tranh giành tự do của Tổ quốc. Vì thế, nghe theo lời kêu gọi của Chủ tịch Hồ Chí Minh, cuối năm 1949, ông đã rời bỏ con đường công danh ở Châu Âu để bí mật trở về nước tham gia kháng chiến.
Từ Châu Âu, ông về Băng Cốc, rồi từ đó qua Campuchia để về Nam Bộ.
Ở Nam Bộ, Giáo sư Lê Văn Thiêm gia nhập Đảng Cộng sản Đông Dương và công tác tại Sở Giáo dục. Ông đã góp phần đào tạo nhiều giáo viên cho vùng kháng chiến. Ít lâu sau, ông lên đường ra Việt Bắc nhận nhiệm vụ mới: lãnh đạo trung tâm đại học đầu tiên của nước Việt Nam dân chủ cộng hoà. Đây thật là một nhiệm vụ quan trọng và phù hợp với khả năng, ý nguyện của ông. Sau 6 tháng gian nan đi bộ từ Nam Bộ lên chiến khu Việt Bắc, Giáo sư Lê Văn Thiêm được giao trọng trách Hiệu trưởng Trường Sư phạm cao cấp và Trường Khoa học cơ bản. Ông đã làm hết sức mình trên cương vị đó, và trở thành người đặt nền móng cho giáo dục đại học của nước Việt Nam mới, người thầy của hầu hết những nhà khoa học Việt Nam được đào tạo trong hơn mươi, mười lăm năm đầu tiên sau cách mạng Tháng Tám.
Từ sau khi hoà bình lập lại, Giáo sư Lê Văn Thiêm được giao nhiều trọng trách: Giám đốc Trường Đại học Sư phạm Khoa học Hà Nội (1954-1956), Phó Hiệu trưởng Trường Đại học Tổng hợp Hà Nội (1957-1970), Viện trưởng đầu tiên của Viện Toán học (1970- 1980). Ông là Đại biểu quốc hội các Khoá II và III. Ông cũng là Đại diện toàn quyền của Việt Nam tại Viện nghiên cứu hạt nhân Đupna, Liên Xô (từ 1956 đến 1980), Chủ tịch đầu tiên của Hội Toán học Việt Nam, Tổng biên tập đầu tiên của hai tờ báo toán học của Việt Nam là Acta Mathematica Vietnamica và Vietnam Journal of Mathematics.
II. Những đóng góp chính về khoa học
Các công trình về lý thuyết Phân phối giá trị các hàm phân hình
Lý thuyết Phân phối giá trị các hàm phân hình (lý thuyết Nevanlinna) được xem là một trong những lý thuyết đẹp nhất của Giải tích toán học Thế kỷ 20. Có thể xem lý thuyết này là sự mở rộng của định lý cơ bản của đại số. Đóng góp quan trọng nhất của Lê Văn Thiêm cho lý thuyết Nevanlinna là kết quả cơ bản về bài toán ngược. Đóng góp quan trọng của Lê Văn Thiêm không chỉ là việc chứng minh sự tồn tại nghiệm của bài toán ngược trong những tình huống tổng quát hơn so với công trình của
Tập 1 Số 3 Tháng 3-2017 Tạp chí Pi 36
Nevanlinna, mà điều quan trọng là lần đầu tiên, ông đã đưa công cụ ánh xạ bảo giác và không gian Teichmuler vào việc giải bài toán ngược. Tư tưởng đó của ông đã được những nhà toán học nổi tiếng khác sử dụng để tiếp tục thu được những kết quả mới cho bài toán ngược: Goldberg, Weitsman, Drasin. Cuối cùng, năm 1977, Drasin cho lời giải trọn vẹn của bài toán ngược của lý thuyết Nevanlinna, 45 năm sau khi bài toán được đặt ra. Điều đáng nói là, trong công trình của mình, Drasin cũng sử dụng những phương pháp mà Lê Văn Thiêm lần đầu tiên áp dụng.
Giáo sư Lê Văn Thiêm ở Thụy Sĩ (1943)
Công trình về bài toán ngược của lý thuyết Nevanlinna đã đặt Lê Văn Thiêm vào hàng ngũ những tác gia kinh điển của lý thuyết này. Cho đến tận ngày hôm nay, hầu như cuốn sách nào về Lý thuyết hàm phân hình, khi nói đến lý thuyết Nevanlinna đều nhắc đến các công trình đầu tiên của Lê Văn Thiêm. Không phải nhà khoa học nào cũng có cái vinh dự được nhắc đến kết quả của mình 60 năm sau! Có thể tin rằng, các công trình đó của Lê Văn Thiêm sẽ còn được nhớ đến nhiều năm, như là một trong những cột mốc của lý thuyết các hàm phân hình.
Trở về Việt Nam năm 1949 theo lời kêu gọi của Chủ tịch Hồ Chí Minh, Giáo sư Lê Văn Thiêm tạm dừng các nghiên cứu khoa học của mình để chuyên tâm vào các nhiệm vụ quan trọng được Nhà nước giao phó. Tuy vậy, khi có chủ trương thúc đẩy phong trào nghiên cứu khoa học trong các trường đại hoc, Giáo
sư lại trở về với lý thuyết diện Riemann yêu thích của mình.
Khi có chủ trương đưa khoa học vào phục vụ thực tiễn sản xuất và chiến đấu, Giáo sư Lê Văn Thiêm chuyển hẳn sang nghiên cứu các vấn đề toán học ứng dụng.
Các công trình về toán học ứng dụng
Vốn là một chuyên gia nổi tiếng về lý thuyết hàm phân hình và diện Riemann, những vấn đề của toán học lý thuyết, Giáo sư Lê Văn Thiêm chuyển sang nghiên cứu và lãnh đạo các nhóm nghiên cứu về toán học ứng dụng. Điều đáng ngạc nhiên là trong số những công trình đầu tiên của ông về toán ứng dụng, có công trình trở thành kinh điển trong lĩnh vực này: lời giải tường minh của bài toán thấm qua hai lớp đất.
Bài toán thấm là vấn đề có ý nghiã thực tiễn quan trọng, xuất hiện khi tính toán sự bền vững của các đê, đập nước, trữ lượng dầu trong các túi dầu, vấn đề rửa mặn các ruộng vùng ven biển,. . .
Trong nhiều bài toán thấm, chẳng hạn khi xét nước thấm qua một con đê dài, ta đi đến mô hình bài toán thấm phẳng (tức là không phụ thuộc một chiều nào đó). Trong thực tiễn, thường có nhiều lớp với hệ số thấm khác nhau nằm dưới một công trình thuỷ lợi: lớp đất sét, lớp đất cát,. . . Đối với trường hợp miền thấm không đồng chất, cho đến trước công trình của Lê Văn Thiêm, người ta chỉ mới có các phương pháp giải gần đúng. Trong công trình Sur un problème d’infiltratione à travers un sol à deux couches. (Về bài toán thấm qua hai lớp đất) đăng trên tạp chí Acta Sci.Vietnam. 1, 1964, pp. 3-9, Lê Văn Thiêm đã dùng Nguyên lý đối xứng trong giải tích phức để xây dựng được nghiệm tường minh cho bài toán thấm qua hai lớp đất với hệ số thấm khác nhau. Đây là công trình đầu tiên trong lĩnh vực lý thuyết nước thấm cho phép xây dựng nghiệm giải tích của bài toán thấm không đồng chất. Điều đó đã được khẳng định trong cuốn sách Lý thuyết chuyển động của nước ngầm của Palubarinova- Kochina xuất bản ở Matxcơva năm 1977.
Tập 1 Số 3 Tháng 3-2017 Tạp chí Pi 37
Một hướng nghiên cứu ứng dụng mà Giáo sư Lê Văn Thiêm cùng các học trò của mình tiến hành trong nhiều năm là nổ định hướng. Phương pháp nổ định hướng do nhà toán học Nga Lavrenchiep đưa ra, dựa trên nguyên tắc sau đây: khi có một vụ nổ lớn, dưới tác động của áp suất quá cao, các vật chất quanh tâm của vụ nổ chuyển động theo quy luật của chất lỏng lý tưởng, tức là không nhớt và không nén được. Chuyển động của chất lỏng lý tưởng có thể mô tả bằng một hàm giải tích.
Nếu tìm được hàm giải tích này, ta có thể tính được áp lực quanh tâm nổ, quỹ đạo chuyển động của vật chất gần tâm nổ. Nhận thấy đây là vấn đề có ý nghĩa thực tiễn lớn, Giáo sư Lê Văn Thiêm đã hướng dẫn các học trò của mình tại Trường đại học Tổng hợp Hà Nội và Viện Toán học nghiên cứu áp dụng. Năm 1966, một nhóm các nhà toán học trẻ của hai cơ quan trên (gồm Ngô Văn Lược, Lê Văn Thành, Nguyễn Văn Lâm, Hà Huy Khoái, Lê Hùng Sơn và một số người khác) lên đường vào Nghệ An để tiến hành trên thực tế. Địa điểm làm việc là vùng Hoàng Mai thuộc địa phận huyện Quỳnh Lưu. Hoàng Mai là nơi gặp nhau của ba tuyến đường vào Nam: đường bộ, đường sắt, đường thuỷ (kênh Nhà Lê). Vì thế, đây trở thành một trong những trọng điểm đánh phá của máy bay Mỹ. Do đường sắt và đường bộ bị hư hại nghiêm trọng, việc vận chuyển qua kênh Nhà Lê trở nên rất quan trọng. Con kênh được đào từ thời Lê, đến nay đã khá cạn. Vấn đề cấp thiết đặt ra là phải nạo vét lòng kênh để các thuyền trọng tải lớn có thể đi qua. Các đơn vị Thanh niên xung phong được giao nhiệm vụ này. Tuy vậy, không thể tập trung một lực lượng lớn, vì máy bay Mỹ bắn phá ngày đêm. Giáo sư Lê Văn Thiêm đề xuất dùng phương pháp nổ định hướng để nạo vét lòng kênh. Mục tiêu đặt ra là làm thế nào để sau khi nổ, hầu hết đất đá văng lên bờ, chứ không rơi lại xuống lòng kênh. Phương pháp nổ định hướng đó cũng được áp dụng trong việc xây dựng các con đường chiến lược trong rừng. Các đơn vị Thanh niên xung phong đã cùng nhóm học trò nói trên của Giáo sư Lê Văn Thiêm áp dụng lý thuyết nổ định hưởng trong việc phá đá, bạt ta-luy, hất những cây to chắn đường xuống vực trong quá trình làm đường.
Sau ngày đất nước thống nhất, Giáo sư Lê Văn Thiêm chuyển vào công tác tại Thành phố Hồ Chí Minh. Ông đã lập nên Phòng Toán học ứng dụng, nghiên cứu các vấn đề toán học đặt ra trong lý thuyết đàn hồi và chuyển động của chất lỏng nhớt.
Các vấn đề toán học ứng dụng mà giáo sư Lê Văn Thiêm quan tâm nghiên cứu đều là những vấn đề được đặt ra trong thực tiễn Việt Nam: xây dựng đê điều và các công trình thuỷ lợi, cải tạo các ruộng nhiễm mặn vùng ven biển, tính toán trữ lượng dầu khí, nạo vét lòng kênh để phục vụ giao thông thời chiến. Ngay khi giải quyết các nhiệm vụ ứng dụng trước mắt, với trình độ cao về khoa học cơ bản, ông đã có những đóng góp quan trọng vào sự phát triển của lý thuyết.
III. Xây dựng nền Toán học Việt Nam
Với những công trình khoa học xuất sắc, Lê Văn Thiêm là người viết trang đầu tiên của lịch sử toán học Việt Nam hiện đại. Ông cũng là một trong những người đầu tiên đặt nền móng xây dựng toán học Việt Nam. Uy tín của ông đã từng là nguyên nhân khiến nhiều thanh niên tài năng lên Chiến khu Việt Bắc để nghiên cứu và giảng dạy toán học: Hoàng Tuỵ, Nguyễn Cảnh Toàn,. . . Và không chỉ lôi cuốn, khuyến khích họ bằng tiếng tăm của mình, Giáo sư Lê Văn Thiêm đã dồn tâm sức để đào tạo lớp thanh niên đầy nhiệt huyết của những ngày đầu cách mạng. “Vốn liếng” của ông khi đó thật ít ỏi, đó chỉ là một ít sách mà ông và một số giáo sư khác đã cố gắng mang theo mình suốt chặng đường từ châu Âu đến chiến khu. Ông luôn khuyến khích những tài năng trẻ đi sâu vào nghiên cứu khoa học, và cố gắng tạo cho họ những điều kiện tốt nhất có thể.
Ngay cả sau khi hoà bình lập lại, các trường đại học ở Việt Nam hầu như chưa có giáo trình đại học về toán bằng tiếng Việt. Vậy mà một trong những quyết tâm lớn của Nhà nước Việt Nam mới là giảng dạy tiếng Việt ở bậc đại học. Lê Văn Thiêm đã dịch và viết các giáo trình, từ Hàm biến phức cho đến Xác suất thống kê. Đến tận năm 1964, chúng tôi vẫn được Thư viện cho mượn các giáo trình do ông dịch, đánh máy bằng tiếng Việt không dấu: có lẽ do thói quen khi còn ở Pháp, hoặc là để tiết kiệm
Tập 1 Số 3 Tháng 3-2017 Tạp chí Pi 38
thời gian khi viết, tiếng Việt của Giáo sư Lê Văn Thiêm thường không có dấu!
Nhận thức rõ tầm quan trọng của Toán học trong việc xây dựng nền khoa học nước nhà, Giáo sư Lê Văn Thiêm cùng với các Giáo sư Tạ Quang Bửu, Hoàng Tuỵ đã vạch một chiến lược lâu dài phát triển Toán học Việt Nam. Sự ra đời của Phòng Nghiên cứu Toán năm 1962 (trực thuộc Uỷ ban Khoa học và Kỹ thuật Nhà nước) là một cột mốc quan trọng trong quá trình xây dựng nền toán học Việt Nam.
Năm 1969, Thủ tướng Phạm Văn Đồng ký quyết định thành lập Viện Toán học thuộc Uỷ ban khoa học và Kỹ thuật Nhà nước. Năm 1970, Giáo sư Lê Văn Thiêm, lúc đó đang là Phó Hiệu trưởng Trường Đại học Tổng hợp Hà Nội, được chuyển về giữ chức vụ Phó Viện trưởng, Phụ trách Viện Toán học. Từ lúc đó, Viện Toán học chính thức đi vào hoạt động.
Với sự lãnh đạo của Giáo sư Lê Văn Thiêm, ngay từ khi thành lập, Phòng Nghiên cứu Toán, và sau này là Viện Toán học đã chú trọng phát triển toàn diện: nghiên cứu cơ bản, nghiên cứu ứng dụng và đào tạo.
Khi chuyển công tác vào Thành phố Hồ Chí Minh, Giáo sư Lê Văn Thiêm đã lập nên Phòng Toán học ứng dụng, với nhiệm vụ nghiên cứu những vấn đề gần với các ứng dụng thực tiễn, đặc biệt là các vấn đề đặt ra tại Miền Nam như thuỷ lợi ở Đồng bằng sông Cửu Long, dầu khí. Giáo sư Lê Văn Thiêm, cùng với Giáo sư Hoàng Tuỵ, là những người đầu tiên gây dựng Khoa Toán của Trường Đại học tổng hợp Hà Nội . Ông luôn kiên trì phương châm giữ vững chất lượng đào tạo, ngay cả trong những năm chiến tranh, khi nhà trường phải sơ tán vào vùng núi Việt Bắc. Ông cũng đã phải trải qua nhiều cuộc đấu tranh gay go trong nội bộ Khoa Toán và Trường Đại học Tổng hợp Hà Nội trong những năm 60 để giữ vững chiến lược đúng đắn đó. Nhờ thế, Khoa Toán của Đại học Tổng hợp Hà Nội (nay là Đại học Khoa học tự nhiên thuộc Đại học Quốc gia Hà Nội) đã đào tạo nên nhiều nhà toán học hàng đầu trong cả nước.
Giáo sư Lê Văn Thiêm
Giáo sư Lê Văn Thiêm cũng là Chủ tịch đầu tiên của Hội Toán học Việt Nam. Với uy tín, tài năng và đức độ của mình, Giáo sư là người lãnh đạo, cũng đồng thời là hạt nhân gắn kết cộng đồng toán học Việt Nam.
Suốt đời hết lòng vì thế hệ trẻ, Giáo sư Lê Văn Thiêm là một trong những người sáng lập tờ báo Toán học và Tuổi trẻ, và trực tiếp viết bài cho báo ngay từ những số đầu tiên. Ông cũng trực tiếp ra đề thi chọn học sinh giỏi toàn Miền Bắc những năm 1963-1964. Ông không nề hà việc gì, dù to dù nhỏ, miễn là có lợi cho việc dìu dắt thế hệ trẻ. Nhiều học sinh giỏi gặp khó khăn khi xét tuyển vào đại học do gia đình, họ hàng bị một số định kiến về “lý lịch” đã tìm đến ông, và được giúp đỡ tận tình. Nhiều người trong số họ đã trở thành những nhà toán học giỏi, có nhiều đóng góp cho đất nước.
Ngay khi cả nước đang trong chiến tranh, máy bay Mỹ bắn phá dữ dội miền Bắc, Giáo sư Lê Văn Thiêm là người đã đứng ra sáng lập tờ báo Toán học và Vật lý bằng tiếng nước ngoài đầu tiên của Việt Nam: tờ Acta Scientiarum Viet- namicarum (Sectio Mathematicarum et Physi- carum). Phần toán học của tờ báo đó ngày nay đã trở thành tờ Acta Mathematica Vietnamica, tờ báo có uy tín nhất về toán của việt Nam, có mặt ở thư viện của nhiều trường
Tập 1 Số 3 Tháng 3-2017 Tạp chí Pi 39
đại học lớn trên thế giới. Việc cho ra đời một tờ báo nghiên cứu toán học (bằng tiếng Anh, Pháp, Nga, Đức) trong chiến tranh là điều hiếm có trên thế giới. Nhiều nhà khoa học nước ngoài đã tỏ ý ngạc nhiên và khâm phục khi thấy Việt Nam, một đất nước đang phải đương đầu với cuộc chiến tranh tàn khốc nhất ở cả hai miền, lại nghĩ đến việc ra một tờ tạp chí nghiên cứu khoa học bằng tiếng nước ngoài. Việc làm đó chứng tỏ tầm nhìn xa của các nhà lãnh đạo khoa học của Việt Nam, và cả sự tin tưởng vào thắng lợi tất yếu của sự nghiệp cách mạng.
Sự phát triển của Toán học Việt Nam, và của khoa học cơ bản Việt Nam nói chung từ sau Cách mạng Tháng Tám mang đậm dấu ấn của Giáo sư Lê Văn Thiêm.
Khó có thể nói hết trong một bài viết ngắn tất cả những gì mà Giáo sư Lê Văn Thiêm đã làm vì sự phát triển một nền Khoa học Việt nam.
Ông thuộc vào số những con người không lặp lại của Lịch sử.
Tập 1 Số 3 Tháng 3-2017 Tạp chí Pi 40
Hà Huy Khoái
I. Về Giải thưởng Lê Văn Thiêm
Giáo sư Lê Văn Thiêm (1918-1991) là chủ tịch đầu tiên của Hội Toán học Việt Nam. Ông là nhà toán học nổi tiếng, có những đóng góp lớn trong nghiên cứu và ứng dụng Toán học. Ông cũng là một trong những người đặt nền móng cho giáo dục đại học ở nước ta, người thầy của nhiều thế hệ các nhà toán học Việt Nam. Giáo sư Lê Văn Thiêm luôn dành sự quan tâm đặc biệt đến việc giảng dạy toán học ở các trường phổ thông. Ông là một trong những người sáng lập hệ thống phổ thông chuyên toán và báo Toán học và Tuổi trẻ.
Giải thưởng Lê Văn Thiêm do Hội Toán học Việt Nam đặt ra nhằm góp phần ghi nhận những thành tích xuất sắc của những thầy cô giáo và học sinh phổ thông đã khắc phục khó khăn để dạy và học toán giỏi, động viên học sinh đi sâu vào môn học có vai trò đặc biệt quan trọng trong sự phát triển lâu dài của nền khoa học nước nhà. Giải thưởng Lê Văn Thiêm cũng là sự ghi nhận công lao của Giáo sư Lê Văn Thiêm, một nhà toán học lớn, một người thầy đã hết lòng vì sự nghiệp giáo dục.
Mỗi năm, Giải thưởng được trao cho một hoặc hai giáo viên THPT có thành tích đặc biệt xuất sắc trong giảng dạy môn Toán, những học sinh đạt thành tích xuất sắc trong các kỳ thi Olympic quốc gia và quốc tế, đồng thời lựa chọn theo học ngành Toán ở bậc đại học (nếu vào năm được giải thưởng đã bước vào đại học). Giải cũng được trao hàng năm cho một học sinh vượt qua nhiều khó khăn, đạt thành tích xuất sắc trong học tập môn Toán. Người
đã được Giải thưởng Lê Văn Thiêm sẽ không được xét trao giải lần thứ hai.
Người được Giải thưởng Lê Văn Thiêm sẽ nhận một Giấy chứng nhận của Hội Toán học Việt Nam, Huy chương Giải thưởng Lê Văn Thiêm, và một khoản tiền mặt (tại thời điểm hiện nay là 5 triệu đồng).
Khoản tiền đầu tiên đóng góp cho Giải thưởng Lê Văn Thiêm được bà quả phụ của Giáo sư Lê Văn Thiêm trích từ tiền Giải thưởng Hồ Chí Minh của Giáo sư. Từ khi ra đời, Giải thưởng Lê Văn Thiêm đã nhận được sự ủng hộ to lớn về tinh thần và vật chất của cộng đồng toán học và toàn xã hội. Đặc biệt, sau dịp kỷ niệm 40 năm Việt Nam tham gia Olympic Toán học quốc tế, một cựu học sinh chuyên toán (đề nghị không nêu tên) đã ủng hộ Quỹ giải thưởng số tiền 1 tỷ đồng.
II. Quy trình xét thưởng
Khác với nhiều Giải thưởng hiện nay ở Việt Nam, ứng viên của Giải thưởng Lê Văn Thiêm không phải viết hồ sơ “xin” giải thưởng. Hội đồng giải thưởng Lê Văn Thiêm, căn cứ quy chế của Giải thưởng, tìm những ứng viên thích hợp trong các giáo viên đang giảng dạy Toán ở các trường phổ thông và các em học sinh. Thông qua nghiên cứu, tìm hiểu, sẽ chọn lọc ra một danh sách ứng viên. Chỉ khi đó, các ứng viên mới được đề nghị gửi hồ sơ để hoàn thiện quy trình xét thưởng. Trên cơ sở đó, Hội đồng xét tặng những người xứng đáng nhất.
Tập 1 Số 3 Tháng 3-2017 Tạp chí Pi 41
III. Giải thưởng Lê Văn Thiêm 2016
Hội Toán học Việt Nam quyết định trao Giải thưởng Lê Văn Thiêm 2016 cho các nhà giáo và học sinh sau đây:
Giáo viên
Thầy giáo Trương Ngọc Đắc, sinh năm 1963, Tổ trưởng tổ Toán trường THPT chuyên Lê Quý Đôn, Bình Định.
Thành tích: 32 năm giảng dạy Toán, trong đó có nhiều năm giảng dạy ở vùng đặc biệt khó khăn. Từ năm học 2000 - 2001 được giao nhiệm vụ phụ trách đội tuyển Toán của Tỉnh, đã đóng góp nhiều công lao tạo nên thành tích của đội tuyển (51 giải toàn quốc, trong đó có 7 giải nhì, 21 giải ba, đặc biệt năm 2016 lần đầu tiên Tỉnh có giải Olympic Toán học quốc tế).
Học sinh
- Hoàng Anh Dũng, học sinh lớp 12, Trường THPT Lam Sơn, Thanh Hoá - Huy chương bạc IMO 2016.
- Lê Nhật Hoàng, học sinh lớp 12, Trường THPT chuyên Lê Quý Đôn, Bình Định - Huy chương bạc IMO 2016. - Phạm Nam Khánh, học sinh lớp 10, Trường THPT Hà Nội - Amsterdam, Hà Nội. Giải nhất cuộc thi Học sinh giỏi toàn quốc năm 2016 (là học sinh lớp 10 đầu tiên đạt thành tích này).
- Phạm Nguyễn Mạnh, học sinh lớp 11, Trường PTNK thuộc ĐHQG TP HCM - Huy chương bạc IMO 2016.
- Đào Vũ Quang, học sinh lớp 12, Trường THPT Hà Nội - Amsterdam, Hà Nội - Huy chương bạc IMO 2016.
- Nông Ngọc Quân, dân tộc Tày, học sinh lớp 11 Trường THPT chuyên Lạng Sơn - Giải khuyến khích kỳ thi học sinh giỏi quốc gia năm 2016.
Buổi lễ trao Giải thưởng Lê Văn Thiêm 2016 đã được tiến hành tại Hà Nội ngày 18/2/2017.
Buổi lễ trao Giải thưởng Lê Văn Thiêm 2016
Tập 1 Số 3 Tháng 3-2017 Tạp chí Pi 42
Giới thiệu trại hè Toán và Khoa học MaSSP
Lê Hồng Nhung* và Đỗ Thị Thu Thảo**
MaSSP (Math and Science Summer Program) là một chương trình hè dành cho học sinh trung học phổ thông (THPT) trên cả nước. Chương trình bắt đầu từ mong muốn được chia sẻ kiến thức và kĩ năng nghiên cứu các môn Toán và Khoa học, của bốn du học sinh ở Mỹ: Đỗ Thị Thu Thảo, Vũ Minh Châu, Nguyễn Trinh và Lê Hồng Nhung. Tham dự chương trình, học sinh có cơ hội học tập và làm việc theo nhóm với các đề tài thú vị, dưới sự hướng dẫn của các anh chị sinh viên, nghiên cứu sinh ở Việt Nam và nước ngoài.
Trại hè Toán và Khoa học MaSSP lần đầu diễn ra vào hai tuần tháng 6 năm 2016, tại Viện Toán học Việt Nam với hai môn, Toán và Tin học. Trong suốt hai tuần, 23 học sinh đến từ 9 tỉnh thành trên cả nước và 10 anh chị hướng dẫn viên (mentor) đã có những ngày học tập, nghiên cứu say mê và hiệu quả. Trong khi các bạn môn Toán tìm tòi những kết quả Toán học thú vị như định lý hôn nhân bền vững, cách tô
màu bản đồ chỉ bằng bốn màu, thì các bạn môn Tin học đã tự mình thiết kế, lập trình ra những trò chơi, ứng dụng, bằng ngôn ngữ Scratch. Hơn thế nữa, các bạn còn được học cách thuyết trình, viết báo cáo, được giao lưu với những khách mời có tên tuổi và chơi nhiều trò chơi lý thú. Đến cuối chương trình, bạn nào cũng cảm thấy yêu thích môn học của mình và tự tin hơn vào bản thân. MaSSP không chỉ là nơi học tập, nghiên cứu, mà còn là một trải nghiệm tuyệt vời, kết nối các bạn học sinh dưới ngôi nhà chung MaSSP.
Cảm nghĩ của một số học sinh tham dự MaSSP 2016:
“MaSSP giúp em tìm lại tình yêu với Toán cũng như tìm thêm nhiều tình yêu mới hết sức đáng yêu.”
“Các anh chị đã cho em biết thế nào là nghiên cứu thật sự và truyền cho em một đam mê vô cùng lớn.”
Tập 1 Số 3 Tháng 3-2017 Tạp chí Pi 43
Đại gia đình MaSSP hè 2016
“MaSSP khác biệt so với các chương trình khác, giúp em định hướng cho tương lai. Môi trường đề cao tự do cá nhân, cho các bạn tự học cái mình muốn, làm cái mình muốn, không bắt ép gì cả, thậm chí em mặc đồ ngủ, tóc tai luộm xuộm đến cũng chả ai nói gì.”
“MaSSP dạy em cách giao tiếp, thuyết trình, từ một người ngại đứng trước đám đông nhưng bây giờ em có thể thuyết trình trước bao nhiêu người chính là vì gia đình MaSSP đã cho em thấy rằng em cũng có thể đứng trước mọi người mà không cầm phải xấu hổ về bản thân, rằng cho dù thế nào cũng luôn có những thành viên của đại gia đình MaSSP ủng hộ em, cổ vũ cho em.”
Năm 2017, ngoài hai môn Toán và Tin học, MaSSP sẽ có thêm hai môn, Hóa học tính toán (Computational Chemistry) và Vật lý, với sự
có mặt của thêm nhiều các anh chị mentor là những người có thành tích cao trong các cuộc thi quốc tế (IMO, IPhO) trước đây và có nhiều kinh nghiệm giảng dạy, nghiên cứu tại các trường đại học danh tiếng trên thế giới, như Viện Công nghệ Mas- sachusetts, đại học Chicago, Princeton, Ecole Polytechnique và Paris 11. Dự kiến chương trình sẽ diễn ra từ ngày 19 tháng 6 đến hết ngày 01 tháng 7 tại Hà Nội. Các học sinh đến từ các khu vực ngoài Hà Nội sẽ được hỗ trợ chi phí đi lại và sinh hoạt, tùy theo nhu cầu.
Chương trình bắt đầu mở đơn từ ngày 15/02/2017 đến ngày 30/03/2017. Thông tin chi tiết về chương trình có ở http://www.masspvn.com và trang facebook https://www.facebook.com/ masspvn. Mọi thắc mắc xin gửi về [email protected].
Tập 1 Số 3 Tháng 3-2017 Tạp chí Pi 44
Nói chuyện với GS Đàm Thanh Sơn
Ngô Bảo Châu (thực hiện)
Giáo sư Đàm Thanh Sơn xuất thân từ một gia đình trí thức truyền thống của Hà Nội. Khi tôi học cấp hai, các thầy cô luôn nêu anh Sơn như một tấm gương thông minh, học giỏi. Sau khi đoạt huy chương vàng IMO với điểm tuyệt đối, anh qua Đại học Tổng hợp Moscow (MGU) học ngành Vật lý lý thuyết. Sau khi tốt nghiệp MGU và bảo vệ luận án tiến sĩ ở Viện Nghiên cứu hạt nhân ở Moscow, anh tiếp tục nghiên cứu và giảng dạy ở MIT (Viện Công nghệ Masachusett), Đại học Columbia (ở New York), Đại học Washington (ở Seattle) và bây giờ ở Đại học Chicago, nơi anh giữ chức danh University Professor là chức danh cao nhất trong hàng giáo sư ở đây.
Giáo sư Đàm Thanh Sơn rất nổi tiếng vì những công trình nghiên cứu chuyên môn sâu cùng với hiểu biết rộng về các chuyên ngành vật lý khác nhau. Độc giả biết tiếng Việt có may mắn được theo dõi những bài viết vừa sâu sắc vừa gần gũi mà anh thường xuyên đăng tải trên blog Đàm Thanh Sơn.
Dưới đây là ghi chép một cuộc nói chuyện với Giáo sư Đàm Thanh Sơn cách đây đã hơn 5 năm, nhưng mỗi lần đọc lại vẫn thấy rất tươi mới. Xin phép tạp chí Pi chia sẻ với độc giả, những người yêu Toán, và chắc chắn cũng yêu việc dùng Toán để hiểu thế giới xung quanh chúng ta.
Tập 1 Số 3 Tháng 3-2017 Tạp chí Pi 45
Ở trường trung học, anh Sơn học chuyên Toán, nhưng khi sang Nga học đại học, anh chuyển sang học Vật lý. Từ lúc còn học trung học, anh đã có định hướng Vật lý chưa? Theo anh, tư duy toán và lý có khác nhau nhiều không?
Thời học phổ thông có một số cuốn sách có ảnh hưởng lớn đến tôi và đến việc chọn đi học Vật lý khi lên đại học. Một cuốn sách là “Vật lý vui”, dịch từ tiếng Nga, tác giả là Yakov Perelman. Cuốn thứ hai là “Câu chuyện về hằng số vật lý cơ bản” của tác giả Đặng Mộng Lân. Ngoài ra, hồi đó tôi còn đặt tạp chí Kvant tiếng Nga, trong đó có rất nhiều bài báo lôi cuốn về Vật lý, viết bởi các nhà khoa học nổi tiếng cho học sinh phổ thông. Bố tôi cũng thích Vật lý, và thỉnh thoảng cũng nói chuyện với tôi về Vật lý.
Vật lý sử dụng rất nhiều công cụ toán. Tôi không nghiên cứu Toán học thật sâu nên không biết chắc chắn có sự khác nhau giữa tư duy toán và tư duy vật lý hay không. Tôi ngờ là có khác nhau, nhưng ít thôi, không nhiều như người ta tưởng. Sự khác nhau lớn nhất có lẽ ở kỳ vọng về kết quả cuối cùng.
Chân lý toán học phải được chứng minh chặt chẽ, chân lý vật lý là phải giải thích được thế giới bên ngoài. Nhiều lúc nó dẫn đến những mối quan tâm khác nhau giữa hai cộng đồng. Thí dụ, một trong những bài toán thiên niên kỷ của Viện Clay là bài toán chứng minh sự
tồn tại của lý thuyết Yang-Mills (và một tính
chất gọi là mass gap của lý thuyết đó). Nhưng các nhà Vật lý từ mấy chục năm nay sử dụng Lý thuyết Yang-Mills để mô tả thế giới các hạt cơ bản một cách rất thành công. Họ mặc nhiên công nhận là Lý thuyết Yang-Mills tồn tại. Điều đó không có nghĩa là không có nhà vật lý nào quan tâm đến việc chứng minh sự tồn tại của Lý thuyết này, nhưng đối với đa số thì vấn đề này không nằm trong danh sách ưu tiên.
Theo anh, thế nào là trực quan vật lý? Liệu có thể các nhà vật lý khác nhau có trực quan vật lý khác nhau hay không? Vai trò của trực quan vật lý trong nghiên cứu khoa học của cá nhân anh Sơn là như thế nào, nó có đối nghịch với tư duy toán học không?
Trực quan vật lý là gì thì rất khó mô tả, nhưng có hai đặc điểm như sau của những người có trực quan vật lý tốt:
– Khả năng ước lượng cỡ độ lớn của các đại lượng.
– Khả năng đơn giản hoá các bài toán bằng cách dùng các phép gần đúng.
Việc trực quan khác nhau giữa những người khác nhau là chuyện rất bình thường. Đó là lý do mà vấn đề cộng tác với các đồng nghiệp là rất quan trọng đối với các nhà vật lý.
Trực quan là cái quan trọng nhất đối với người làm Vật lý. Đôi khi trực quan có thể đánh lừa ta. Điều này rất dễ xảy ra khi ta
Tập 1 Số 3 Tháng 3-2017 Tạp chí Pi 46
nghiên cứu thế giới lượng tử, hay thế giới tương đối tính, vì trong cuộc sống hàng ngày ta chỉ tiếp xúc với những vật cổ điển, chuyển động chậm. Để phát triển trực giác lượng tử, trực giác tương đối tính ta phải làm việc trong thế giới đó một thời gian. Trực quan thường không đối nghịch với tư duy toán học, mà bổ sung cho tư duy toán học. Khi trực quan và toán học mâu thuẫn với nhau thì chắc chắn có vấn đề với một trong hai cách tư duy. Những lúc đó có thể có những vấn đề hay và mới nảy sinh.
Đối tượng nghiên cứu của Vật lý lý thuyết hình như là những gì mình không nhìn thấy bằng mắt, không nghe thấy bằng tai. Liệu nhà vật lý, hay cụ thể là cá nhân anh Sơn, có cần quan sát thế giới xung quanh nữa không?
Đúng là Vật lý lý thuyết hiện nay là một môn học khá trừu tượng. Nhưng về bản chất, Vật lý là một khoa học thực nghiệm. Tôi không tin rằng Vật lý lý thuyết có thể phát triển không có thực nghiệm. Tất cả sự giàu có của Vật lý đều từ thế giới bên ngoài mang lại. Nếu tự nhiên Vật lý lý thuyết bị tách rời khỏi thế giới xung quanh thì tôi chắc rất nhiều người sẽ bỏ ngành đó đi làm việc khác. Cá nhân tôi không làm thực nghiệm, nên việc quan sát thế giới bên ngoài phải làm qua các nhà vật lý thực nghiệm.
Trong những công trình khoa học của mình, anh Sơn thấy cái gì là tâm đắc hơn cả? Liệu anh có thể giải thích để những người dốt Vật lý như tôi hiểu được không?
Hiện nay tôi đang tâm đắc về một bài báo tôi viết hơn một năm trước đây với một sinh viên người Ba Lan (Piotr Surówka) về vấn đề phân chia phải trái trong thủy động lực học tương đối tính. Tôi tâm đắc một phần là vì vấn đề này có thể giải thích bằng một ngôn ngữ tương đối dễ hiểu. Đại khái là, nếu ta có một chất lỏng làm từ các hạt quark, và cho nó quay xung quanh một trục, thì các quark xoáy trái và các quark xoáy phải sẽ hơi bị lạng sang hai chiều khác nhau trên trục quay.
Để dễ hình dung hơn, ta có thể lấy một thí dụ khá gần gũi với cuộc sống. Ta nhớ lại, Louis Pasteur phát hiện ra là phân tử của nhiều chất không có đối xứng gương. Phân tử đường ta
uống là như vậy. Đường nguồn gốc sinh học theo quy định gọi là đường tay phải. Nếu tổng hợp, ta sẽ có đường cả hai loại, tay trái và tay phải. Các hạt quark, nếu chuyển động với tốc độ gần bằng tốc độ ánh sáng, cũng có thể phân thành hai loại như vậy.
Giả sử ta hòa tan một thìa đường tay trái và một thìa đường tay phải vào một cốc nước. Bây giờ ta quay cái cốc nước với một vận tốc góc nhất định. Nếu các phân tử đường cũng xử sự giống như các hạt quark, thì các phân tử
đường tay trái sẽ hơi nổi lên trên, các phân tử đường tay phải sẽ hơi chìm xuống dưới, hoặc ngược lại. Trên thực tế, hiện tượng này không xảy ra với các phân tử đường, nhưng nó lại xảy ra với các hạt quark.
Sự khác biệt giữa hai trường hợp liên quan đến những vấn đề trừu tượng như dị thường lượng tử (quantum anomaly), Lý thuyết Chern-Simons, và tôpô. Tôi cũng hy vọng hiện tượng này có thể quan sát được trong thực nghiệm.
Tôpô là bộ phận toán học chỉ quan tâm đến dạng của hình thể mà không quan tâm đến kích thước. Đối với tôpô, phần của mặt phẳng giới hạn bởi một đường cong không tự cắt bất kỳ, không khác gì hình tròn. Tôi rất tò mò làm sao các thuộc tính tô pô lại có thể có ích trong việc tìm hiểu thế giới vật lý mà như anh Sơn nói, cái trực quan cơ bản là ước lượng độ lớn của các đại lượng?
Đúng là các đại lượng vật lý bao giờ cũng được đo bằng số. Thế nhưng, có những đại lượng chỉ nhận được những giá trị gián đoạn; trong nhiều trường hợp lý do liên quan đến tôpô. Ví dụ, nhiều hợp kim ở trạng thái siêu dẫn (tức là ở nhiệt độ rất thấp) có một tính chất rất lạ. Khi ta cho nó vào từ trường đủ mạnh thì từ trường thâm nhập vào chất siêu dẫn, nhưng số đường sức từ trường thâm nhập vào không phải là tùy ý, mà phải là một số nguyên lần một đại lượng gọi là lượng tử của từ thông. Tại sao lại như vậy? Giải thích ra thì dài dòng, nhưng cuối cùng đó là do π1(S1) = ℤ. Có nhiều ví dụ khác tương tự như vậy. Trong một mô hình (gọi là mô hình Skyrme) sự bền vững của hạt proton là do π3(S3) = ℤ. Rất lạ là việc vật chất xung quanh ta không
Tập 1 Số 3 Tháng 3-2017 Tạp chí Pi 47
phân rã đi thành các dạng năng lượng khác liên quan đến π3(S3) = ℤ!
Được biết anh Sơn có quan tâm đến âm nhạc, xin hỏi anh một câu về đề tài này. Vật lý có giải thích được tại sao có những hòa âm ta thấy rất chói tai, mà có những hòa âm lại thật dịu dàng, có hòa âm nghe lần đầu thì êm tai, nghe vài lần thì khó chịu như ăn quá nhiều thịt mỡ, còn có những hòa âm thì mỗi lần nghe lại, lòng ta vẫn không khỏi bồi hồi?
Câu hỏi của bạn đi xa khỏi ranh giới của Vật lý rất nhiều, và chắc là đi ra ngoài cả ranh giới của khoa học tự nhiên nữa. Pythagoras thường được coi là người đầu tiên khám phá ra mối liên hệ giữa Âm nhạc và Toán học: các hòa âm nghe êm tai có tỷ lệ giữa các tần số của các nốt gần với các phân số tối giản. Nhưng tại sao có những hợp âm gây buồn, gây vui, tôi không giải thích được.
Theo anh Sơn thì phải làm gì để những nhà vật lý trẻ ở Việt Nam có thể bám sát vào dòng chảy chính của Vật lý thế giới?
Vấn đề này không đơn giản, tôi phải suy nghĩ thêm. Trên thế giới nhiều nước (như Hàn quốc, Brazil) đã giải quyết vấn đề này khá thành công. Trước mắt ta có thể học hỏi kinh nghiệm của họ.
Anh đã từng học và làm Vật lý ở Moscow, Boston, New York, Seattle và có thể nhiều nơi
khác nữa. Anh Sơn có thấy cái cách người ta học và làm Vật lý ở những địa điểm khác nhau, có gì khác nhau không?
Cũng khó nói. Ở mỗi chỗ khác nhau tôi làm những vấn đề khác nhau, có những kỷ niệm khác nhau. Những cá nhân tôi làm việc chung cũng có những tính cách rất đa dạng. Nhưng nếu nhìn tổng thể thì sự nghiên cứu vật lý ở các nơi tôi đã ở không khác nhau nhiều lắm. Còn về cách học thì có lẽ cách học ở Nga có hơi khác ở Mỹ. Lúc tôi học ở Nga thì tôi được tương đối tự do, không bị kiểm tra nhiều và nhiều môn lên lớp cũng không bắt buộc, miễn là qua được kì thi cuối học kỳ là được. Tôi có rất nhiều thời gian để đọc sách.
Ở Nga có một sự tách biệt giữa giảng dạy và nghiên cứu. Giảng dạy tập trung ở trường đại học, và nghiên cứu tập trung ở các Viện. Điểm này là một điểm yếu của hệ thống Nga.
Theo anh Sơn để trở thành một nhà vật lý thực thụ có những phẩm chất gì? Những phẩm chất đó phải được rèn luyện trong những hoàn cảnh như thế nào?
Có lẽ trong ngành khoa học nào cũng vậy, muốn thành công ít nhất phải có hai kỹ năng: tìm ra vấn đề hay, và giải quyết được vấn đề.
Liệt kê được hết những điều kiện để rèn luyện ra những kỹ năng đó thì chắc là dài, sợ viết hết ra thì thành cliché.
Tập 1 Số 3 Tháng 3-2017 Tạp chí Pi 48
APMO DÀNH CHO HỌC SINH TIỂU HỌC VÀ THCS (APMOPS) Trần Nam Dũng
Hiện nay có nhiều kỳ thi quốc tế dành cho học sinh Tiểu học và Trung học cơ sở đã được các trường đại học, các Trung tâm giáo dục tổ chức tại Việt Nam, như APMOPS (Singapore), AMC (Mỹ), AMC (Australia), IMC (Singapore), AMO (Mỹ-Singapore), IKMC (Cuộc thi toán Kanga- roo, quốc tế), ITOT (Cuộc thi toán giữa các thành phố, Nga), . . . Điều này đã tạo cho các em học sinh nhiều cơ hội thi đấu và cọ xát. Trong số các kỳ thi vừa nêu, APMOPS là kì thi có mặt ở Việt Nam sớm nhất.
Olympic Toán học khu vực châu Á - Thái Bình Dương dành cho học sinh Tiểu học và Trung học cơ sở (tiếng Anh: Asia-Pacific Mathemat ical Olympiad for Primary Schools; viết tắt là APMOPS) là kỳ thi Toán học do Học viện Hwa Chong (Singapore) tổ chức hằng năm với sự tham gia của học sinh (Tiểu học và Trung học cơ sở) đến từ các quốc gia và vùng lãnh thổ thuộc khu vực châu Á - Thái Bình Dương. Kỳ thi gồm hai vòng và thi hoàn toàn bằng tiếng Anh. Vòng Một gồm 30 bài toán với yêu cầu chỉ ghi đáp số, thời gian làm bài là 120 phút. Vòng Hai gồm 6 bài toán tự luận, làm trong 120 phút. Những thí sinh đạt kết quả vòng Một cao nhất (thông thường phải đoạt giải Bạch kim, giải cao nhất của APMOPS) sẽ được mời tham dự vòng Hai tại Singapore. Các thí sinh tham dự vòng Hai được tài trợ toàn bộ chi phí đi lại, ăn ở.
APMOPS được tổ chức lần đầu tiên vào năm 1990. Việt Nam bắt đầu tham gia APMOPS từ
năm 2009. Qua 8 lần tham dự, trừ kỳ thi đầu mang tính thử nghiệm, các học sinh Việt Nam luôn đạt được kết quả tốt tại cả hai vòng thi, luôn đạt đủ cơ số huy chương ở vòng Một và tỷ lệ học sinh lọt vào Top 40 ở vòng Hai luôn đạt mức cao.
Các bài toán vòng Một của APMOPS không quá khó, nhưng đòi hỏi tư duy Toán học tốt, hiểu tình huống và yêu cầu của đề bài, giải quyết nhanh chóng vấn đề đặt ra (trung bình 6 phút/một bài toán). Ở vòng một, thí sinh không cần trình bày lời giải, chỉ cần ghi đáp số (và đáp số sai tất nhiên không được điểm). Các đề Toán khá đa dạng về kiến thức nền và hấp dẫn ở cách phát biểu. Chính vì điều đó, học sinh sẽ rất hứng thú khi học tập, khi thi thử và khi đi thi. Ở vòng Hai, mức độ khó sẽ cao hơn nhiều và thí sinh sẽ phải làm 6 bài toán tự luận (trình bày đầy đủ lời giải) trong vòng 120 phút.
Với chất lượng đề thi tốt và ổn định qua các năm, APMOPS được đánh giá là kỳ thi uy tín trong các kỳ thi quốc tế và khu vực; các học sinh được huy chương APMOPS có thêm ưu thế khi dự xét tuyển học bổng. Nhiều học sinh đoạt huy chương APMOPS sau này tiếp tục gặt hái những thành công ở những bậc học tiếp theo.
Theo Điều lệ của APMOPS, các học sinh có tuổi dưới 12 tính đến ngày thi đều có thể dự thi. Vì thế, các học sinh lớp 6, thậm chí học sinh lớp 7
Tập 1 Số 3 Tháng 3-2017 Tạp chí Pi 49
(sinh sau tháng 4) trong hệ thống giáo dục Việt Nam đều có thể dự thi. Trước năm 2015, một học sinh có thể tham dự nhiều lần (tại Việt Nam có học sinh tham dự 2 lần và cả hai lần đều đạt huy chương Bạch Kim), nhưng từ năm 2015, Điều lệ chỉ cho phép mỗi học sinh tham dự một lần.
Tại Việt Nam, APMOPS được tổ chức tại các cơ sở của Tập đoàn giáo dục quốc tế Kinder World Vietnam. Trước năm 2015, kỳ thi được tổ chức song song tại 3 địa điểm: Hà Nội, Đà Nẵng, Bình Dương, nhưng kể từ năm 2015, BTC Việt Nam đã rút gọn lại còn một địa điểm duy nhất tại trường Quốc tế Việt Nam Singapore, Cầu Giấy, Hà Nội.
Cuối cùng, chúng tôi xin điểm qua một số kiến thức thường được sử dụng trong các bài toán thi APMOPS để bạn đọc hình dung, cũng như giới thiệu một số bài toán hay trích từ các đề thi AP- MOPS qua các năm, để chúng ta cùng thư giãn trí tuệ. Các mảng kiến thức hàm chứa trong các bài toán thi bao gồm: Số học (số nguyên, phân số), Đại số (tỉ số phần trăm, tỷ lệ
thuận, tỷ lệ nghịch, phương trình bậc nhất), Hình học (diện tích các hình), Logic-IQ (quy luật dãy số, dãy hình), Tổ hợp (Nguyên lý Dirichlet (còn được gọi là Nguyên lý “chuồng và thỏ ”), các bài toán đếm đơn giản, xác suất đơn giản).
Để tham dự kì thi, học sinh cần được trang bị những suy luận logic căn bản, rèn luyện cảm nhận về số, kỹ năng phát hiện quy luận, tư duy quy nạp (không hoàn toàn); phần Hình học, cần được trang bị công thức tính diện tích các hình, cần biết cách sử dụng phương pháp diện tích để tính các tỷ lệ, định lý Thales, định lý Pythagore; . . . Đặc biệt cần được bổ sung kiến thức cơ bản và kỹ năng giải các bài toán đếm bằng cách sử dụng các quy tắc đếm (cộng, nhân, trừ), sử dụng phương pháp liệt kê trường hợp, biết cách sử dụng biểu đồ Venn, nguyên lý “chuồng và thỏ” ở mức đơn giản. Học sinh cũng cần được làm quen với xác suất ở mức độ sơ cấp nhất của khái niệm này (ví dụ, tung một con xúc xắc cân đối, đồng chất thì xác suất ra mặt 6 là 1/6).
Đa số các bài toán thi của APMOPS là các bài toán với yêu cầu chỉ ghi đáp số (dạng toán chứng minh chỉ có thể xuất hiện ở vòng Hai);
để giải nhanh và đúng, cần những suy luận thông minh, những cách mô hình hoá hợp lý, hơn là kỹ năng tính toán và biến đổi tốt. Dưới đây là một số bài toán, trích từ các đề thi APMOPS các năm vừa qua:
1. Đường kính thẳng đứng của một hình tròn dịch sang phải 3cm, đường kính nằm ngang dịch lên trên 2cm như hình vẽ dưới đây:
Hãy tìm hiệu diện tích phần gạch chéo và diện tích phần không gạch của hình tròn.
2. Cỏ ở một cánh đồng sẽ mọc thêm với tốc độ không đổi mỗi ngày. 200 con cừu sẽ ăn hết đồng cỏ trong 100 ngày. 150 con cừu sẽ ăn hết đồng cỏ trong 150 ngày. Hỏi 100 con cừu sẽ
ăn hết đồng cỏ trong bao nhiêu ngày?
3. Mỗi cạnh của hình ngũ giác ABCDE được tô bởi một trong 3 màu: đỏ, vàng và xanh. Hỏi có bao nhiêu cách tô 5 cạnh của ngũ giác sao cho hai cạnh kề nhau khác màu?
4. Có 9 tấm bìa, được đánh số từ 1 đến 9. Bốn người A,B,C,D, mỗi người chọn cho mình 2 tấm bìa.
A nói: “Tổng hai số của tôi bằng 6”.
B nói: “Hiệu hai số của tôi bằng 5”.
C nói: “Tích hai số của tôi bằng 18”.
D nói: “Một trong hai số của tôi gấp đôi số kia”.
Hỏi tấm bìa còn lại có số nào?
5. Bốn đội bóng A,B,C và D thi đấu trong một bảng. Hai đội bất kỳ đấu với nhau đúng một
Tập 1 Số 3 Tháng 3-2017 Tạp chí Pi 50
trận. Trong một trận, đội thắng được 3 điểm, đội thua được 0 điểm, còn nếu trận đấu có kết cục hoà thì mỗi đội được 1 điểm. Sau khi tất cả các trận đấu kết thúc người ta nhận thấy: (1) tổng điểm của các đội là các số lẻ liên tiếp ; (2) D có tổng điểm cao nhất ; (3) A có đúng hai trận hoà, trong đó có một trận hoà với C. Tìm tổng điểm của mỗi đội bóng.
6. Tam giác vuông trong hình vẽ sau đây được đặt sao cho một cạnh góc vuông nằm trên một đường thẳng (vị trí (1)). Nó xoay quanh điểm A để đến vị trí (2). Sau đó, nó lại xoay quanh điểm B đến vị trí (3). Cuối cùng, nó xoay quanh điểm C để đến vị trí (4). Biết rằng AP = BP = CP = 10cm, tìm tổng chiều dài đường đi mà điểm P đã vẽ nên. (Lấy π = 3.14).
7. David muốn đi từ lầu 2 xuống lầu 1 ở một trung tâm mua sắm. Nếu anh ấy đi xuống 14 bậc, khi sử dụng thang cuốn, thì anh ấy có thể đi từ trên xuống dưới trong 30 giây. Nếu anh ấy đi xuống 28 bậc thì anh ấy sẽ xuống đến dưới sau 20 giây. Tính số bậc của thang cuốn.
8. ABE và BCD là các tam giác vuông. D nằm trên cạnh BE sao cho AE = ED = DC = 1cm và DB = 2cm. Tính góc theo đơn vị độ.
9. Trong hình vẽ dưới đây, diện tích của tam giác ABC bằng 5cm2 , AE = ED và BD = 2DC. Hãy tính diện tích phần tô đậm.
10. Gọi Sn là tổng các chữ số của số nguyên dương n, ví dụ S3 = 3, S29 = 2 + 9 = 11. Hãy tính tổng S1 + S2 + S3 +···+ S2010 + S2011.
Tập 1 Số 3 Tháng 3-2017 Tạp chí Pi 51
Thế nào là MỘT NĂM?
Phạm Vũ Lộc
Trước hết, hãy tìm hiểu nguồn gốc của đơn vị đo thời gian quen thuộc là năm. Cũng như các đơn vị để tính lịch khác, năm cũng có nguồn gốc tự nhiên trước khi được con người thể chế hóa vào lịch ở các nền văn hóa khác nhau trong lịch sử. Tất nhiên, một bạn học sinh cũng có thể trả lời khái quát rằng một năm là “thời gian Trái Đất quay một vòng quanh Mặt Trời”. Nhưng mọi chuyện không đơn giản như vậy.
Nếu nhìn từ Trái Đất, Mặt Trời di chuyển hết một vòng trên Hoàng đạo từ Tây sang Đông thì tức là Trái Đất trong thực tế đã quay hết một vòng quanh Mặt Trời. Khi Mặt Trời trở lại vị trí ban đầu so với nền sao trên thiên cầu, thì nó kết thúc một chù kỳ gọi là năm sao, dài
365.25636 ngày (ở đây dùng đơn vị ngày SI dài 86,400 giây).
Tuy nhiên, con người có ý niệm về chu kỳ năm lại bắt nguồn từ nhận thức về chu kỳ mùa. Chúng ta đã biết hiện tượng mùa là do địa trục nghiêng so với mặt phẳng quỹ đạo Trái Đất, nên Xích đạo trời nghiêng so với Hoàng đạo trên thiên cầu.
Mặt Trời di chuyển trên Hoàng đạo trong một năm từ điểm Xuân Phân (đi lên qua Xích đạo trời) tới Hạ Chí (xa nhất trên Thiên cầu Bắc), rồi Thu Phân (đi xuống qua Xích đạo trời), đến Đông Chí (xa nhất trên Thiên cầu Nam). Vị trí của Mặt Trời trong từng giai đoạn tương ứng với các mùa trên Trái Đất (xem hình trên). Nếu chỉ như vậy thì chu kỳ mùa cũng là năm sao. Tuy nhiên, do lực hấp dẫn chủ yếu từ Mặt
Tập 1 Số 3 Tháng 3-2017 Tạp chí Pi 52
Trăng và Mặt Trời, địa trục lắc trong không gian tạo thành một hình nón, giống như con quay, với chu kỳ khoảng 25.771 năm, gọi là hiện tượng tiến động (hình dưới).
Do đó, Thiên cực Bắc chạy trên một đường tròn, còn Xích đạo trời xoay trượt đi so với Hoàng đạo, làm điểm Xuân Phân cũng như ba điểm còn lại chạy trên Hoàng đạo với cùng chu kỳ, theo chiều từ Đông sang Tây, ngược chiều đi của Mặt Trời. Hệ quả là Mặt Trời đến điểm Xuân Phân hàng năm sớm hơn khoảng 20 phút 24 giây so với năm sao. Người ta gọi chu kỳ này là năm xuân phân hay năm thời tiết, là cơ sở của phép làm lịch, dài trung bình 365.24219 ngày∗. Vì hiện tượng này xảy ra rất từ từ nên phải qua thời gian dài, các nền văn minh khác nhau mới lần lượt nhận ra lịch tính theo năm sao bị sai so với thời tiết, gọi là hiện tượng tuế sai, và tìm cách hiệu chỉnh lại lịch.
Dương lịch ta dùng hiện nay là lịch Gregory (do giáo hoàng Gregory XIII ban bố năm
1582) có độ dài năm dân sự là 365.25 ngày. Theo phép lịch này, cứ 400 năm (146,097 ngày) là một chu kỳ, ngày tháng và thứ trong tuần lại trùng nhau. Chia ra thì mỗi năm trung bình dài 365.24250, đã rất gần với giá trị năm thời tiết trung bình ở trên, với sai số là thừa 1 ngày sau hơn 3000 năm.
Ngoài ra, còn một chu kỳ năm khác cũng đáng chú ý trong thiên văn học để tính chu kỳ nhật thực và nguyệt thực (gọi chung là thiên thực) là năm thiên thực. Nó được định nghĩa là chu kỳ giữa hai lần Mặt Trời đi qua tiết điểm† của Bạch đạo. Bạch đạo chính là quỹ đạo Mặt Trăng quanh Trái Đất chiếu lên thiên cầu, nghiêng so với Hoàng đạo khoảng 5.145◦. Khi Mặt Trăng trên Bạch đạo tới gần Hoàng đạo vào lúc trăng non thì có thể xảy ra nhật thực và vào lúc trăng rằm thì có thể xảy ra nguyệt thực. Dưới tác dụng của lực hấp dẫn từ Trái Đất, quỹ đạo Mặt Trăng tiến động với chu kỳ 6793.4765 ngày hay gần 18.6 năm làm hai tiết điểm của nó chạy trên Hoàng đạo với chiều từ Đông sang Tây, ngược chiều đi của Mặt Trời. Như vậy là năm thiên thực trung bình cũng ngắn hơn năm sao và bằng 346.62008 ngày‡.
Trong văn hóa Ấn Độ, hai tiết điểm của Mặt Trăng lên và xuống so với Hoàng đạo là hai thiên thể vô hình có khả năng nuốt Mặt Trăng và Mặt Trời, gọi là Rahu và Ketu. Chúng được truyền tới Trung Hoa và sau đó là Việt Nam với tên phiên âm lại là La Hầu và Kế Đô, kết hợp cùng Mặt Trăng, Mặt Trời và năm hành tinh, tạo thành Cửu Diệu trong chiêm tinh học phương Đông.
* Độ dài năm xuân phân không đều nhau vì trong khi tốc độ tiến động của địa trục dù có thể coi là ổn định (do chỉ xét 1 năm trong chu kỳ 25.771 năm), thì tốc độ của Mặt Trời trên Hoàng đạo lại không đều vào các thời điểm khác nhau trong năm, do quỹ đạo Trái Đất là hình elip nên quay quanh Mặt Trời lúc nhanh lúc chậm. Hết một năm xuân phân, Mặt Trời lại gặp điểm xuân phân đi ngược lại nên tổng quãng đường cả hai đi được là một vòng trọn vẹn (năm sao): năm xuân phân trung bình + (năm xuân phân trung bình)/(chu kỳ tiến động điểm xuân phân) = năm sao. Từ đó tính ra được năm xuân phân trung bình dài 365.24219 ngày, ngắn hơn năm sao khoảng hơn 20 phút.
†Tiết điểm: Giao điểm của quỹ đạo thiên thể với mặt phẳng tham chiếu, ở đây là mặt phẳng hoàng đạo.
‡Tương tự với cách tính năm xuân phân trung bình, ở đây điểm xuân phân được thay bằng tiết điểm Mặt Trăng.
Tập 1 Số 3 Tháng 3-2017 Tạp chí Pi 53
Trò chơi SET và Toán học
Phạm Nguyễn Minh Đức∗ và Đỗ Thị Thu Thảo∗∗
Trò chơi SET
Trò chơi SET được thiết kế bởi Marsha Falco
vào năm 1974 và được xuất bản bởi công ty
SET Enterprises vào năm 1991. Hiện nay, trò
chơi này rất phổ biến ở Mĩ, cũng như ở một số
nước khác trên thế giới.
Một bộ bài SET gồm các lá bài đôi một khác
nhau; trong mỗi lá bài:
– Có đúng một trong ba loại hình vẽ: hình thoi,
hình dấu ngã, hình con nhộng;
– Các hình vẽ có đúng một trong 3 màu: xanh, đỏ, tím;
– Phần trong của các hình vẽ được trình bày theo đúng một trong ba kiểu: để rỗng, tô đặc, gạch sọc;
– Số lượng các hình vẽ là 1, 2 hoặc 3.
Như vậy, mỗi lá bài đều có 4 yếu tố cấu thành: loại hình vẽ, màu sắc của hình vẽ, họa tiết phần trong của hình vẽ (dưới đây gọi tắt là họa tiết) và số lượng hình vẽ. Vì với mỗi yếu tố cấu thành đều có 3 phương án lựa chọn nên tổng số lá bài của một bộ bài SET bằng 81 (= 34) (xem Hình 1 dưới đây).
Hình 1: Tất cả các lá bài của một bộ bài SET. Nguồn: Internet
Một bộ ba lá bài được gọi là một SET nếu ở mỗi yếu tố cấu thành (trong 4 yếu tố nêu trên), ba lá bài hoặc là giống nhau hết hoặc là đôi một khác nhau.
Hình 2: Mỗi bộ 3 lá bài ở hàng trên là một SET, hàng dưới không là một SET. Nguồn: Internet
Tập 1 Số 3 Tháng 3-2017 Tạp chí Pi 54
Ở Hình 2, ta thấy bộ ba lá bài thứ nhất (ở hàng trên) là một SET, vì ba lá bài đó đôi một khác nhau ở mỗi yếu tố cấu thành. Bộ ba lá bài thứ hai ở hàng trên cũng là một SET, vì ba lá bài đó đôi một khác nhau ở hai yếu tố, loại hình vẽ và họa tiết, đồng thời chúng hoàn toàn giống nhau ở hai yếu tố còn lại, màu sắc và số lượng hình vẽ. Bộ ba lá bài ở hàng dưới không phải là một SET, vì ở yếu tố họa tiết, chúng không giống nhau hoàn toàn, cũng không đôi một khác nhau.
Có nhiều cách chơi bộ bài SET; cách phổ biến nhất là: Khai cuộc, người điều hành bày ra bàn 12 lá bài, những người chơi quan sát và nếu ai phát hiện ra SET trong 12 là bài đó thì lấy SET ấy cho mình (trường hợp có 2 hay nhiều người hơn cùng nhìn thấy một SET, ai nhanh tay hơn người đó sẽ lấy được). Giả sử có k SET (trong 12 lá bài ban đầu) được lấy đi. Khi đó, người điều hành sẽ bày tiếp ra bàn 3k lá bài và những người chơi lại làm như vừa mô tả. Quá trình cứ thế được lặp lại cho tới khi ở trên bàn có 12 lá bài nhưng không người chơi nào lấy đi SET nào (do không có hoặc do người chơi không phát hiện ra); lúc này, người điều hành sẽ lần lượt bày tiếp ra bàn từng bộ 3 lá bài cho tới khi có người chơi lấy đi các SET; sau khi các SET được lấy đi, nếu số lá bài có trên bàn lớn hơn 12 thì người điều hành sẽ lặp lại các thao tác vừa nêu, còn nếu số lá bài có trên bàn nhỏ hơn 12 thì người điều hành sẽ bày bổ sung các lá bài mới sao cho số lá bài có trên bàn vừa đúng là 12. Cuộc chơi cứ như thế tiếp diễn cho tới khi tất cả 81 lá bài đã được bày ra trên bàn. Người thắng trong cuộc chơi là người lấy được cho mình nhiều SET hơn cả.
Tưởng như SET chỉ là một trò chơi mang tính giải trí đơn thuần. Ấy thế nhưng, những người yêu Toán lại nhìn thấy ở trò chơi này rất nhiều vấn đề toán học hay và thú vị; từ những tính chất, câu hỏi xác suất, tổ hợp đơn giản, cho đến những câu hỏi hóc búa làm khó bao nhiêu nhà toán học. Trong bài viết này, chúng ta sẽ cùng điểm qua một vài vấn đề trong số ấy, từ dễ đến khó.
Một số tính chất toán học thú vị
Trong khuôn khổ bài viết này, chúng tôi xin được giới thiệu 3 tính chất toán học thú vị liên quan đến trò chơi SET.
Tính chất 1: Nếu chọn ngẫu nhiên ba lá bài thì khả năng ba lá bài này tạo thành một SET là .
Tính chất 2: Số loại SET, mà ba lá bài trong mỗi SET đôi một khác nhau ở đúng 3 yếu tố, là nhiều nhất.
Tính chất 3: Số lá bài tối thiểu cần bày ra bàn để đảm bảo luôn tìm được một SET là 21.
Trước khi đọc tiếp, các bạn hãy thử tự mình chứng minh những tính chất nêu trên xem sao. Tính chất nào dễ thấy và tính chất nào không hiển nhiên chút nào? Để chứng minh Tính chất 1, bạn phải để ý rằng với hai lá bài bất kì sẽ tồn tại duy nhất một lá bài để hợp với hai lá kia tạo thành một SET. Do đó, tổng số SET có trong bộ bài bằng số bộ 2 lá bài phân biệt chia cho 3 (vì mỗi bộ 3 lá bài có 3 lựa chọn cho hai lá bài “nguồn”), tức là SET. Lấy số này chia cho tổng số bộ 3 lá bài có thể có (là tổ hợp chập 3 của 81 và bằng ta tìm được xác xuất để 3 lá bài được chọn ra ngẫu nhiên là một SET chính là . Từ đây ta có thể tính được kì vọng số SET trong 12 lá bài được lấy ra ngẫu nhiên là , và nhiều tính chất khác liên quan tới xác suất.
Mất thêm vài phút, các bạn cũng có thể chứng minh được Tính chất 2. Ba lá bài trong một SET có thể đôi một khác nhau ở một, hai, ba hoặc cả bốn yếu tố cấu thành. Để đếm số SET mà ba lá bài trong SET đôi một khác nhau ở cả 4 yếu tố (ví dụ như bộ ba lá bài thứ nhất ở
Hình 2), ta lần lượt chọn các lá bài: Lá bài thứ nhất có 81 cách chọn. Lá bài thứ hai, do khác với lá bài thứ nhất ở cả 4 yếu tố nên chỉ có 24 = 16 cách chọn. Lá bài cuối cùng, theo phần chứng minh ở trên, chỉ có đúng 1 cách chọn.
Tập 1 Số 3 Tháng 3-2017 Tạp chí Pi 55
Như vậy sẽ có = 216 SET thuộc loại này. Ở đây ta phải chia cho 3! vì SET không tính đến thứ tự chọn các lá bài. Bằng lý luận tương tự, ta sẽ tìm được số SET mà ba lá bài trong SET đôi một khác nhau ở đúng 3, 2, 1 yếu tố, tương ứng, là 432, 324, 108. Từ đó suy ra số SET mà ba lá bài trong SET đôi một khác nhau ở đúng 3 yếu tố là lớn nhất. Do đó, một chiến thuật khi chơi là nhắm tới việc tìm các SET mà ba lá bài trong SET đôi một khác nhau ở đúng 3 yếu tố, sau đó tới đôi một khác nhau ở đúng 2 yếu tố, vì khả năng xuất hiện của chúng lớn hơn cả.
Còn Tính chất 3 thì sao? Chúng tôi tin chắc rằng bạn không thể tìm ra lời giải trong vòng vài phút. Thật vậy, đây là một kết quả khó, cần xét rất nhiều trường hợp và người ta tìm ra lời giải nhờ vào sự hỗ trợ của máy tính. Câu hỏi hóc búa và thú vị này là khởi đầu cho rất nhiều nghiên cứu toán học mà chúng ta sẽ đề cập ở mục tiếp theo.
Mô hình toán học và kết quả đột phá gần đây
Để tìm hiểu các tính chất của trò chơi SET, các nhà toán học đã mô hình hóa trò chơi bằng không gian của trường hữu hạn, kí hiệu là F3, gồm 3 phần tử, mà ta có thể tưởng tượng là 0,1,2. Ta có thể cộng trừ nhân chia các phần tử này theo modulo 3. Ví dụ 2+1 = 0 và 2×2 = 1. Không gian bốn chiều của trường hữu hạn này, kí hiệu là F 3 4 là một không gian mà mỗi điểm được xác định bởi 4 tọa độ, mỗi tọa độ là một phần tử trong F3 Bây giờ ta có thể coi mỗi lá bài trong trò chơi SET là một điểm trong không gian này, với tọa độ thứ nhất chỉ loại hình vẽ (0 = hình thoi, 1= dấu ngã, 2 = con nhộng), tọa độ thứ hai chỉ màu sắc của hình vẽ (0 = xanh, 1 = đỏ, 2 = tím), tọa độ thứ ba chỉ họa tiết (0 = rỗng, 1 = tô đặc, 2 = gạch sọc) và tọa độ cuối cùng chỉ số lượng hình vẽ (0 = 1 hình, 1= 2 hình, 2 = 3 hình). Ví dụ: lá bài đầu tiên trong Hình 2 là điểm (2, 1, 1, 2) vì hình vẽ là con nhộng (2), màu đỏ (1), tô đặc (1) và có 3 hình (2).
Trong mô hình nêu trên, ba lá bài tạo thành một SET khi và chỉ khi ba điểm tương ứng với chúng, giả sử là (x1, x2, x3, x4),(y1, y2, y3, y4) và
(z1,z2,z3,z4) có tính chất: xi + yi + zi = 0,i = 1,4. Thật vậy, do ở mỗi yếu tố cấu thành, ba lá bài tạo thành một SET hoặc giống nhau hoàn toàn hoặc đôi một khác nhau nên (xi , yi ,zi) ∈ {(0,0,0),(1,1,1),(2,2,2),(0,1,2)},i = 1,4; từ đây hiển nhiên suy ra điều vừa nêu. Ta gọi 3 điểm có tính chất như vậy là phụ thuộc nhau, hoặc cũng có thể nói chúng thẳng hàng với nhau trong không gian . Như thế, một SET chính là một đường thẳng, hay một tập phụ thuộc. Khái niệm thẳng hàng ở đây phải được hiểu theo nghĩa rộng hơn trong hình học Euclid thông thường, tuy nhiên cũng có nhiều điểm tương đồng với khái niệm thẳng hàng trong hình học Euclid. Ví dụ như trong hình học Euclid, nếu ta xét các điểm trong hệ tọa độ Descartes thì ba điểm có tổng các tọa độ bằng 0 sẽ thẳng hàng với nhau. Ví dụ khác là, qua hai điểm bất kì tồn tại đúng một đường thẳng chứa cả hai điểm; tương ứng với nó, từ hai lá bài bất kì ta sẽ tìm được đúng một SET. Đây chính là nhận xét quan trọng để tìm ra Tính chất 1.
Như đã đề cập ở mục trước, một trong những câu hỏi hóc búa và nhận được nhiều sự quan tâm nhất là số lá bài nhiều nhất sao cho trong đó không có một SET nào hết. Câu hỏi này tương ứng với việc tìm kích cỡ lớn nhất của một tập các điểm trong không gian sao cho không có 3 điểm nào thẳng hàng. Ta gọi một tập như thế là tập độc lập (cap set). Ta có thể mở rộng khái niệm này cho không gian n chiều với n là một số tự nhiên lớn hơn 1. Ta kí hiệu số phần tử lớn nhất của một tập độc lập trong là rn. Giá trị này có thể được tính trực tiếp cho các số n nhỏ nhờ sự trợ giúp của máy tính. Ví dụ như r4 = 21 theo Tính chất 3. Vì việc tính giá trị chính xác của rn là vô cùng khó nên điều mà các nhà toán học quan tâm hơn cả là sự tăng trưởng của rn theo n.
Liệu ta có thể có rn > c3n, với c là một hằng số nào đó không? Nói một cách khác, liệu có tồn tại một tập độc lập với số phần tử tỉ lệ với tổng số điểm trong không gian hay không? Từ lâu, các nhà toán học đã chứng minh được điều này là không thể. Bằng cách sử dụng giải tích điều hòa, vào năm 1985 Meshulam đã
Tập 1 Số 3 Tháng 3-2017 Tạp chí Pi 56
chứng minh được . Phải mất gần 30 năm sau, năm 2012, Bateman và Katz mới tìm ra một kết quả tốt hơn , với ε là một số dương nhỏ tùy ý. Tuy không tốt hơn là mấy, đây đã được coi là một bước tiến đáng kể, và chứng minh của nó khá phức tạp, dài mấy chục trang.
Tháng 5 năm 2016, một bước đột phá trong bài toán này làm bất ngờ cộng đồng toán học. Ngày 5 tháng 5 năm 2016, Croot, Lev và Pach đã đăng tải trên arXiv (thư viện mở của đại học Cornell, nơi lưu trữ rất nhiều bài báo toán và các môn khoa học khác) một bài báo về một bài toán tương tự. Bài báo ngay lập tức thu hút sự chú ý của nhiều nhà toán học trong lĩnh vực này. Chỉ sau đó vài ngày, dựa vào một bổ đề quan trọng trong bài báo này, hai nhà toán học Jordan S. Ellenberg và Dion Gijswijt độc lập chứng minh được rn = O(2.756n) (theo 5), có nghĩa là một tập độc lập phải có ít hơn 3n phần tử rất nhiều. Các bạn có thể thử một vài giá trị của n để thấy được 2.756n tăng trưởng chậm hơn nhiều thế nào. Điều bất ngờ nhất là chứng minh này chỉ vỏn vẹn trong 2 trang giấy và không sử dụng bất kì kiến thức cao siêu nào. Công cụ chính là đại số tuyến tính, một chút kiến thức về đa thức và xác suất ngẫu nhiên, mà bất kì sinh viên năm đầu đại học nào cũng có thể hiểu được. Với sức mạnh của Internet, tin tức nhanh chóng lan truyền trong cộng đồng toán học. Một loạt blog toán nổi tiếng như blog của Terrence Tao, Tim Gowers, Gil Kalai, Jordan Ellenberg... đã đưa tin về chứng minh mới này. Chỉ sau hai tuần, trên arXiv xuất hiện một loạt các bài báo mới về việc sử dụng kĩ thuật mới này để giải quyết các bài toán khác, như giả thuyết hoa hướng dương của Erdos-Szemerédi, một bài toán nổi tiếng trong lý ̈thuyết tập hợp về một họ tập hợp giao nhau theo hình hoa hướng dương (xem [4]). Kĩ thuật mới này cũng được dùng để loại bỏ một cách hiệu quả một thuật toán nhân ma trận (xem [1]). Như tờ báo Quanta
Magazine đã viết, chứng minh này làm bàng hoàng các nhà toán học (xem [5]).
Toán học còn rất nhiều những điều kì diệu đang chờ ta khám phá. Là một nghiên cứu sinh, đôi khi tôi (Đỗ Thị Thu Thảo) cảm thấy choáng ngợp về sự phát triển chóng mặt của Toán học, khi mà để hiểu được một bài toán hay kết quả nghiên cứu nào đó, ta phải đọc rất nhiều kết quả trước đấy, phải học đủ những kiến thức này nọ. Chứng minh của bài toán tập độc lập (the cap set prob- lem) là một minh chứng hùng hồn rằng đâu đây vẫn có những chứng minh đẹp và sơ cấp đang chờ ta khám phá.
Cuối cùng, các tác giả của bài viết này xin chân thành cảm ơn các thành viên Trại hè MaSSP, đặc biệt là bạn Hồ Đức (Đại học Chicago), về sự giúp đỡ, về những góp ý quý báu cho việc hoàn thành bài viết.
Tài liệu tham khảo
[1] Jonah Blasiak, Thomas Church, Henry Cohn, Joshua A. Grochow, Eric Naslund, William F. Sawin, Chris Umans. On cap sets and the group-theoretic approach to ma- trix multiplication. arXiv: 1605.06702, May 2016.
[2] Ernie Croot, Vsevolod Lev, Peter Pach. Progression-free sets in Z n 4 are exponen tially small. arXiv: 1605.01506, May 2016.
[3] Jordan S. Ellenberg and Dion Gijswijt. On large subsets of Fnq with no three-term arith metic progression. arXiv: 1605.09223, May 2016.
[4] Gil Kalai. Polymath 10 Emergency Post 5: The Erdos-Szemerédi Sunflower Conjecture ̋is Now Proven. http://gilkalai.wordpress.com, May 2016.
[5] Erica Klarreich. Simple Set Game Proof Stuns Mathematicians. Quanta Magazine, May 2016.
Tập 1 Số 3 Tháng 3-2017 Tạp chí Pi 57
Số nhà
Phan Thanh Hồng và Nguyễn Thị Nhung
Thấy Nam bò ra sàn, tính tính toán toán, Tuấn hỏi:
- Bảo đến đón nàng đi shopping cơ mà, sao còn ngồi đây?
Nam vò đầu, bứt tai, trả lời:
Minh họa: Tuấn Linh
- Lần trước ghi số nhà nàng vào điện thoại, lỡ tay xóa mất, giờ không nhớ, mà nàng lại mới mất điện thoại, hẹn hôm nay cùng đi mua, nên không gọi hỏi được.
- Sao tìm số nhà mà lại tính toán gì nhiều thế? Pi thắc mắc.
- Chả là thế này, lần trước đến nhà nàng, nàng chưa về nên ngồi nói chuyện với bố nàng. Đang không biết nói gì, thấy có tiếng trẻ con nô đùa ngoài vườn, tớ hỏi: “Trẻ con chơi ngoài kia là cháu của nhà mình hết ạ?”. Bố nàng nói: “Cậu cứ đùa, lương dạy toán đâu đủ nuôi nhiều con cháu thế!”. Thấy bị hớ, tớ gỡ gạc: “Thế các anh chị nhà mình được mấy cháu rồi ạ?”. Là một giáo viên dạy chuyên Toán nên bố nàng đã trả lời thế này: “So với ba nhà xung quanh (nhà ông An, nhà ông Hùng, nhà ông Lâm), số cháu nhà tôi nhiều nhất, rồi đến nhà ông An, đến nhà ông Hùng, còn nhà ông Lâm có ít nhất. Cả thẩy có không đến 18 đứa trẻ và tích của số đứa trẻ trong bốn gia đình đúng bằng số nhà của nhà tôi, mà cậu đã nhìn thấy khi vào nhà”. Hôm đó, tính toán một lúc, để xác định số trẻ của mỗi nhà, tớ phải hỏi thêm là có phải nhà ông Lâm có ít nhất 2 đứa trẻ
không. Khi bố nàng trả lời xong, tớ đã nói chính xác số trẻ của từng nhà. Bố nàng có vẻ hài lòng lắm, nên sau lần đó thái độ của nàng khác hẳn. Giờ đang ngồi tính để suy lại số nhà nàng, mà chưa ra.
Tuấn nghe xong, liền tươi cười bảo Nam:
- Nếu tớ giúp cậu tìm ra số nhà nàng thì cậu tính sao?
Nam mừng quá, bảo: “Cậu làm ơn tính giúp đi, muốn gì tớ cũng xin hậu tạ”.
- Ok, không đòi gì nhiều. Tý đưa nàng về, qua nhà sách Mụ Hoa mua cho tớ tờ Pi là được.
- Tưởng gì, chuyện nhỏ. Mua cho 10 tờ luôn, hôm trước đi qua thấy ở đó đang treo biển “Giảm giá hết cỡ” mà.
Các bạn nhanh tay tính số nhà để được tặng nhiều báo Pi “trong mùa giảm giá” nào.
Giải đáp bài: Lọ Lem và dì ghẻ (Xem Tạp chí Pi số 1 Tập 1 năm 2017)
Đã tới giờ “đóng cửa” mà vẫn chưa thấy bạn nào gửi lời khuyên cho Lọ Lem nên Chủ quán đành alo hỏi Pi vậy.
Minh họa: Linh Rab
Pi đã bày cho Lọ Lem cách biết tủ nào đựng gì chỉ bằng một lần mở khóa! Vấn đề là ở chỗ Lọ Lem có thông tin tất cả các tủ đang bị dán sai nhãn. Pi bảo Lọ Lem mở một ngăn tùy ý của tủ có dán nhãn “Hạt dẻ + Đỗ đen”. Giả sử ngăn đó
Tập 1 Số 3 Tháng 3-2017 Tạp chí Pi 58
đựng đỗ đen (trường hợp đựng hạt dẻ được suy luận tương tự). Khi đó sẽ biết ngay ngăn còn lại trong tủ này cũng là đỗ đen, vì nếu ngược lại thì tủ này được dán nhãn đúng. Như vậy đã tìm được tủ có hai ngăn đều đựng đỗ đen. Từ đó suy ra được luôn tủ có dán nhãn
“Hạt dẻ” là tủ có một ngăn đựng đỗ đen, một ngăn đựng hạt dẻ, vì tủ này không thể có hai ngăn đều đựng đỗ đen, do tủ có hai ngăn đỗ đen đã vừa tìm được rồi. Tủ còn lại hiển nhiên là tủ có hai ngăn đựng toàn hạt dẻ.
Chủ quán
Tập 1 Số 3 Tháng 3-2017 Tạp chí Pi 59
Que tính và các con số, phép tính
Vũ Anh Tuấn
Chúng ta đã quen thuộc với các chữ số 0, 1, 2,
3, 4, 5, 6, 7, 8, 9 được tạo nên từ các nét cong.
Tuy nhiên, chữ số điện tử xuất hiện ở khắp
mọi nơi (trên bảng hiệu quảng cáo, đồng hồ
báo tín hiệu giao thông, máy tính bỏ túi, đồng hồ điện tử, máy đếm ngược thời gian,. . . ) lại được tạo nên từ các đoạn thẳng!
- Được phép xếp các số 1 và 3 theo các mẫu sau:
- Trong một phép tính, trong một số thì các
chữ số phải có chiều cao tương đương nhau,
phải được xếp cùng một kiểu. Chẳng hạn,
không được xếp số 41 như hình dưới đây:
Trong bài viết này, mời các bạn thử sức với một vài câu đố về các con số (số điện tử, số La Mã) được xếp bằng các que tính. Trước tiên, chúng ta cần thống nhất với nhau về quy tắc xếp các số Ả-Rập bằng các que tính:
- Các số 0, 2, 4, 5, 6, 7, 8 và 9 được xếp theo mẫu sau:
Dưới đây, chúng ta cùng xem xét một số bài tập, các bạn nhé.
Bài 1:
a) Với 3 que tính bạn có thể xếp được những số nào? Số lớn nhất xếp được là bao nhiêu?
b) Có 5 que tính, bạn có thể xếp được số lớn nhất trong hệ thập phân là mười một nghìn một trăm mười một (11111). Đố bạn, trong các số xếp được, số lớn nhất trong phạm vi 100 là số nào?
Tập 1 Số 3 Tháng 3-2017 Tạp chí Pi 60
Trả lời:
a) Với 3 que tính ta có thể xếp được nhiều số. Chúng ta có thể xếp được các số Ả-Rập: bảy, một trăm mười một, và một loạt các số La Mã: ba, bốn, sáu, chín, mười một, một trăm.
Hình 1: Số bảy và số một trăm mười một trong hệ thập phân.
Hình 2: Các số La Mã: ba, bốn, sáu.
Hình 3: Các số La Mã: chín, mười một, một trăm.
Từ đó, số lớn nhất mà tác giả đã xếp ra được là một trăm mười một (trong hệ thập phân).
b) Với 5 que tính, các số (trong hệ thập phân) trong phạm vi 100 có thể xếp được là: 2; 5; 10; 13; 17; 31; 71. Do đó, số lớn nhất trong phạm vi 100 có thể xếp được là 71.
Bài 2: Cho phép tính sai được xếp bởi các que
- Thay đổi các con số.
Kết hợp những điều trên với lưu ý rằng, số 9 có thể bỏ đi một nét để thành số 5 và số 5 có thể thêm vào một nét để trở thành số 6, tìm ta ra cách chuyển 1 que tính để có được phép tính đúng như sau:
Câu đố cho các bạn:
1) Cho phép tính sai:
Di chuyển đúng hai que tính, hãy làm phép tính trên trở thành phép tính đúng.
2) Xếp số 351 theo hình dưới đây:
Bằng cách di chuyển đúng hai que tính, hãy tạo ra số lớn nhất có thể.
3) Chỉ di chuyển một que tính, hãy làm cho phép tính với các số La Mã sau trở thành phép tính đúng:
tính:
4) Cho phép tính sai:
Chỉ di chuyển 1 que tính, hãy làm cho phép tính trên trở thành phép tính đúng.
Hướng dẫn:
Với bài toán này, các khả năng thay đổi mà chúng ta có thể nghĩ đến gồm:
- Thay đổi phép toán;
- Thay đổi vị trí dấu bằng;
Di chuyển đúng 3 que tính, hãy làm phép tính trên trở thành phép tính đúng.
5) Dậu xếp phép tính:
Tập 1 Số 3 Tháng 3-2017 Tạp chí Pi 61
Phép tính trên là một phép tính sai. Dậu đố Thân di chuyển một que tính để có được một phép tính đúng. Thân nghĩ mãi chưa ra, bạn có thể giúp Thân?
Giải đáp câu đố trong bài “Que tính kì diệu”
(Xem Tạp chí Pi số 1 Tập 1 năm 2017)
1) Nếu số que tính ít hơn 3, không thể xếp được tam giác. Nếu số que tính bằng 3, xếp được đúng 1 tam giác.
Trường hợp có 4 que tính, có thể xếp được 4 tam giác; chẳng hạn, như hình dưới đây:
Vậy, cần ít nhất 4 que tính để có thể xếp được số tam giác bằng với số que tính.
Tập 1 Số 3 Tháng 3-2017 Tạp chí Pi 62
THỂ LỆ GỬI BÀI
• Bài gửi email đến [email protected] cần ghi rõ ở Chủ đề (Subject) của email: Lời giải của bài toán nào; Bài toán đề xuất; Bài viết cho chuyên mục gì; hoặc Bài viết không theo chuyên mục nào.
• Trên trang đầu bản thảo từng bài toán, bài viết cần ghi rõ họ tên tác giả, địa chỉ thư tín, email và số điện thoại để tiện liên hệ.
• Các bài toán đề xuất cần kèm theo lời giải và gợi ý trước về khối lớp. Bản dịch hay sưu tầm cần ghi rõ nguồn.
• Bản thảo gửi theo email với định dạng latex hoặc word, cùng với file định dạng pdf.
• Tạp chí chỉ nhận những bài chưa đăng trên báo trong và ngoài nước.
ĐỊA CHỈ GỬI BÀI
• Qua email: [email protected]
• Qua bưu điện đến:
Tòa soạn Tạp chí Pi
P705 - B8, Tầng 7, Tòa nhà Thư viện Tạ Quang Bửu, Trường Đại học Bách khoa Hà Nội,
Số 1 Đại Cồ Việt, Q. Hai Bà Trưng Hà Nội.
THỂ LỆ ĐẶT BÁO
Bạn đọc quan tâm có thể đặt báo theo một trong các hình thức sau:
• Đặt tại tất cả các bưu cục trong cả nước. Mã của Tạp chí Pi là C057.
• Đặt tại website www.pi.edu.vn: chọn nút Đặt tạp chí và làm theo hướng dẫn.
• Đối với Tòa soạn gửi email đến địa chỉ [email protected] và điền vào form trên thư trả lời của Tòa soạn. Thanh toán vào tài khoản của Tạp chí Pi:
Tài khoản số: 0851000012293
Ngân hàng Vietcombank
Chi nhánh Dịch vụ khách hàng đặc biệt
(trong nội dung chuyển tiền cần ghi rõ họ tên, điện thoại người chuyển) Sau khi thanh toán xong gửi email đến Tòa soạn để được xác nhận. • Đặt mua bản điện tử tại Alezaa.com
Tập 1 Số 3 Tháng 3-2017 Tạp chí Pi 63
BAN BIÊN TẬP
Hà Huy Khoái (Tổng Biên tập)
Trần Nam Dũng (Phó Tổng Biên tập)
Nguyễn Thị Lê Hương (Thư ký Tòa soạn)
Nguyễn Khắc Minh (Trưởng ban Biên tập)
Ngô Bảo Châu, Phạm Huy Điển, Nguyễn Thành Nam, Trần Văn Nhung, Nguyễn Duy Thái Sơn, Chu Cẩm Thơ, Ngô Việt Trung, Vũ Hà Văn, Nguyễn Ái Việt, Lê Anh Vinh.
CỘNG TÁC VIÊN THƯỜNG XUYÊN
Hạ Vũ Anh, Võ Quốc Bá Cẩn, Phạm Ngọc Điệp, Lưu Thị Thanh Hà, Trần Minh Hiền, Phan Thanh Hồng, Trần Quang Hùng, Hà Duy Hưng, Đậu Hoàng Hưng, Nguyễn Văn Huyện, Vũ Thế Khôi, Nguyễn Duy Liên, Phạm Vũ Lộc, Trần Quốc Luật, Kiều Đình Minh, Nguyễn Thị Nhung, Đặng Văn Sơn, Đỗ Hoàng Sơn, Lê Xuân Sơn, Lưu Bá Thắng, Đào Mạnh Thắng, Nguyễn Tất Thu.
TÒA SOẠN
Tầng 7 Thư viện Tạ Quang Bửu, Trường Đại học Bách khoa Hà Nội Đường Đại Cồ Việt, P. Bách khoa, Quận Hai Bà Trưng, Hà Nội
Email gửi bài: [email protected]. Giao dịch: [email protected]
Điện thoại: 04.3215.1407 Fax: 04.3623.1543
Website: www.pi.edu.vn
Chế bản: Lê Quốc Tuấn, Trần Minh Tâm
Đồ họa: Nguyên Trần, Đoàn Xuân Trường
Ảnh bìa 1: Lê Văn Thiêm
(Nguồn: Viện Toán học)
Giấy phép số 484/GP-BTTTT ngày 25.10.2016
In tại: Công ty Cổ phần In Hà Nội
Tạp chí ra hàng tháng
Giá: 25.000 đồng
Tập 1 Số 3 Tháng 3-2017 Tạp chí Pi 64
Tập 1 Số 3 Tháng 3-2017 Tạp chí Pi 65